SlideShare ist ein Scribd-Unternehmen logo
1 von 62
Downloaden Sie, um offline zu lesen
MRCP Infectious Disease
Notes
BY DR. SHERIF ELBADRAWY
MRCP Infectious Disease Revision notes
Dr.Sherif Elbadrawy
Causes of Bacterial meningitis ? - Streptococcus pneumoniae
- Neisserie meningitidis
- Listeria rnonocytogenes
Causes of Otitis media ? - Streptococcus pneumonia
Causes of Community-acquired-
Pneumonia ?
- Streptococcus pneumonae
- Haemophitus influenzae
- Staphylococcus aureus
Causes of Atypical Pneumonia ? - Mycoptasma pneumoniae
- Chlamydia pneumoniae
- Legioneiia pneumophiia
Causes of Eye infections ? - Staphylococcus aureus
- Neissoria gonorrhoeae
- Chlamydia trachomatis
MRCP Infectious Disease Revision notes
Dr.Sherif Elbadrawy
Dr. Sherif
Elbadrawy
Digitally signed by Dr. Sherif Elbadrawy
DN: cn=Dr. Sherif Elbadrawy, o=Westmead
Hospital, ou,
email=sherif_badrawy@yahoo.com, c=AU
Date: 2017.01.18 19:30:37 +11'00'
Causes of Sexually transmitted
diseases ?
- Chlamydia trachomatis
- Neisseria gonorrhoeae
- Treponema pallidum
- Ureaplasma urealyticum
- Haemophilus ducreyi
Causes of Skin infections ? - Staphylococcus aureus
- Streptococcus pyogenes
- Pseudomonas aeruginosa
Causes of Urinary tract infections ?
- Escherichia col
- Other Enterobactedaceae
- Staphylococcus saprophyticus
- Pseudomonas aeruginosa
Gram (P)ositive cocci ? sta(P)hylococci + stre(P)tococci
(including enterococci)
Gram (N)egative cocci ? (N)eisseria meningitidis +(N)eisseria
gonorrhoeae, also Moraxella
MRCP Infectious Disease Revision notes
Dr.Sherif Elbadrawy
Gram positive bacilli ?
☀ Actinomyces
☀ Bacillus antracis (anthrax)
☀ Clostridium
☀ Diphtheria: Corynebacterium
diphtheriae
☀ Listeria monocytogenes
Antibiotics with anti-anaerobic
activity ?
☆ Penicillins
☆ Cephalosporins (except ceftazidime)
☆ Metronidazole
☆ Clindamycin
☆ Erythromycin
☆ Tetracycline
Antibiotics with NO anti-anaerobic
activity ?
☆ Gentamicin
☆ Ciprofloxacin
☆ Ceftazidime
Incubation Period Longer than 3
weeks ?
☆ Infectious mononucleosis
☆ Cytomegalovirus
☆ Viral hepatitis
☆ HIV
Incubation Period 2 - 3 weeks ? ☆ Mumps
☆ Rubella
☆ Chickenpox
MRCP Infectious Disease Revision notes
Dr.Sherif Elbadrawy
Incubation Period 1 - 2 weeks ?
☆ Malaria
☆ Measles
☆ Dengue fever
☆ Typhoid
Incubation Period Less than 1 week ?
☆ Scarlet fever
☆ Influenza
☆ Diphtheria
☆ Meningococcus
Vaccine using Fragment/Extracts of
the organism or virus ?(recombinant
DNA technology)
✰ diphtheria
✰ pertussis ('acellular' vaccine)
✰ heptitis B
✰ meningococcus, pneumococcus,
hemophilus
Composition of cholera Vaccine ?
inactivated Inaba and Ogawa strains of
Vibrio cholerae together with
recombinant B-subunit of the cholera
toxin
Post-Exposure Prophylaxis in
Hepatitis A ?
IV IG or hepatitis A vaccine
MRCP Infectious Disease Revision notes
Dr.Sherif Elbadrawy
Post-Exposure Prophylaxis in
Hepatitis B ?
☠ HBsAg positive source :-
✾ a known responder to HBV vaccine→ a booster dose should be given.
✾ If in the process of being vaccinated or are a nonresponder→
IV IG + vaccine
☠ Unknown source :-
✾ a known responder to HBV vaccine→a booster dose of
HBV vaccine.
✾ If a nonresponder→ IV IG + vaccine
✾ If in the process of being vaccinated →an accelerated course of HBV
vaccine
Post-Exposure Prophylaxis in
Hepatitis C ?
Monthly PCR - if seroconversion ➜
interferon +/- ribavirin
Post-Exposure Prophylaxis in HIV ?
✿ A combination of oral antiretrovirals (e.g.
Tenofovir, emtricitabine, lopinavir and ritonavir)
ASAP (i.e. Within 1-2 hours, but may be started up to
72 hours following exposure) for 4 weeks
✿ Serological testing at 12 weeks following
completion of post-exposure prophylaxis
✿ ↓ risk of transmission by 80%
Post-Exposure Prophylaxis in
Varicella zoster ?
✿ Antibody Testing
✿ VZIG for IgG negative pregnant
women /immunosuppressed
Indictions of Tetanus vaccine ?
✿ given in the UK as part of the routine
immunisation
☀ high-risk wounds (e.g. compound fractures, delayed
surgical intervention, severely devitalised tissue) :-
✿ Vaccinated➜ IM IG.
✿ Incomplete or unknown vaccination history ➜ IM
IG + a dose of tetanus vaccine.
MRCP Infectious Disease Revision notes
Dr.Sherif Elbadrawy
Indictions of Meningococcal Vaccine
?
Meningococcal vaccine is routinely indicated at the age 11
visit. The vaccine is also indicated for adults with the
following circumstances:
• Asplenia
• Terminal complement deficiency
• Military recruits
• Residents of college dormitories
• Travelers to Mecca or Medina in Saudi Arabia for the
Hajj (pilgrimage)
Tetanus ➜ Causative organism,Mode
of infection, mechanism of toxicity ?
Clostridium tetani➜Tetanus spores
in soil → introduced into the body
from a wound➜tetanospasmin
exotoxin➜prevents release of GABA
Clinical picture of Tetanus ?
❒ Prodrome fever, lethargy, headache
❒ Trismus (lockjaw)
❒ Risus sardonicus
❒ Opisthotonus (arched back,
hyperextended neck)
❒ Spasms (e.g. Dysphagia)
Rx of Tetanus ?
❁ MV & ms relaxants
❁ IM IG. for high-risk wounds (e.g.
Compound fractures, severely
devitalised tissue)
❁ Metronidazole is DOCH
When to suspect HIV seroconversion
?
☹ a glandular fever 3-12 weeks after infection
☹ symptomatic in 60-80% of patients (Man returns
from trip abroad with maculopapular rash and flu-
like illness - think HIV seroconversion).
☹ For questions involving businessmen always
consider sexually transmitted infections.
MRCP Infectious Disease Revision notes
Dr.Sherif Elbadrawy
Clinical picture of HIV
seroconversion (glandular fever)?
☆ HIV seroconversion resembles infection with cytomegalovirus,
toxoplasmosis and the Epstein-Barr virus
◥ Lymphadenopathy
◥ Diarrhoea
◥ Maculopapular rash
◥ Sore throat
◥ Malaise, myalgia, arthralgia
◥ Mouth ulcers
◥ Rarely meningoencephalitis
Dx of HIV seroconversion ?
◥ ELISA, confirmed by Western blotting
◥ Antibodies to HIV may not be present
◥ HIV PCR and p24 antigen tests can
confirm diagnosis
◥ The CD4+ count is not used for
diagnostic purposes
immunological changes in
progressive HIV ?
❀ Reduction in CD4 count
❀ Increase B2-Microglobulin (IBM)
❀ Decrease IL-2 production (DIL=DELL) 〘
IBM & DELL 〙
❀ Polyclonal B-cell activation
❀ ↓ NK cell function
❀ ↓ delayed hypersensitivity responses
Contraindicated Vaccines in HIV +ve
pt ? (Cholera
SABIN's Beautiful, INcredible!) (all
are live attenuated except for
cholera)
❀ Cholera
❀ SABIN's = Sabin's polio virus
(oral)
❀ B = BCG
❀ IN = influenza (intranasal)
Vaccines that can be used if CD4 >
200 in HIV +ve pt ? (Varicella Yellow
MMR)
❀ Varicella
❀ Yellow Fever
❀ MMR
MRCP Infectious Disease Revision notes
Dr.Sherif Elbadrawy
Causes of Diarrhea in HIV +ve pt ?
❂ CRYPTOSPORIDIUM (most common)+
other protozoa.
❂ Effects of the virus itself (HIV enteritis)
❂ Cytomegalovirus
❂ Mycobacterium avium intracellulare
❂ Giardia
Cryptosporidium infection in HIV
+ve pt ?
☆ MCC of DIARRHEA in HIV patients.
☆ incubation period of 7 days.
☆ Ziehl-Neelsen stain of the stool ➜
characteristic red cysts of
Cryptosporidium.
☆ Difficult to Rx (supportive therapy)
Mycobacterium avium intracellulare
infection in HIV +ve pt ?
❐ CD4 < 50.
❐ fever, sweats, abdominal pain &
DIARRHEA ± hepatomegaly and ↑LFTs
❐ Dx ➜ blood cultures and BM aspiration
❐ Rx ➜ rifampicin, ethambutol and
clarithromycin
Factors which ↓ vertical transmission
of HIV ?
✪ Maternal antiretroviral therapy
✪ Neonatal antiretroviral therapy
✪ Mode of delivery (caesarean
section)
✪ Infant feeding (bottle feeding)
Should pregnant women do
screening for HIV ?
【YES】
NICE guidelines recommend offering
HIV screening to all pregnant women
MRCP Infectious Disease Revision notes
Dr.Sherif Elbadrawy
Whom should be offered
antiretroviral therapy in pregnant
HIV +ve pt ?
✪ 【ALL】pregnant HIV +ve pt
✪ If not using before ➜ start at 28
and 32 weeks of gestation ➜
continue intrapartum.
Mode of delivery in pregnant HIV
+ve pt ?
✪ Elective CS
✪ zidovudine infusion before CS
✪【Zidovudine 】 orally to the
neonate for 4 to 6 weeks.
✪ AVOID breast feed
Best Neonatal antiretroviral therapy
to protect against HIV of the baby in
HIV +ve mother ?
✪【Zidovudine 】 orally to the
neonate for 4 to 6 weeks.
✪ AVOID breast feed
Etiology of Kaposi's sarcoma ?
HHV-8 (Human Herpes Virus 8)
【purple papules】 or plaques on the:-
✿ skin➜ ± ulcerate
✿ mucosa (e.g. GIT and respiratory tract)➜ massive
hemoptysis and pleural effusion
✿ HAART➜ ↑CD4 count ➜ improves Kaposi's sarcoma
✿ Radiotherapy + Surgical resection may be used
✿ Intralesional injections of vincristine or interferon
Clinical picture of Kaposi's sarcoma ?
【purple papules】 or plaques on the:-
✿ skin➜ ± ulcerate
✿ mucosa (e.g. GIT and respiratory
tract)➜ massive
hemoptysis and pleural effusion
MRCP Infectious Disease Revision notes
Dr.Sherif Elbadrawy
Rx of Kaposi Sarcoma ?
✿ HAART➜ ↑CD4 count ➜ improves
Kaposi's sarcoma
✿ Radiotherapy + Surgical resection may
be used
✿ Intralesional injections of vincristine or
interferon
most common opportunistic infection
in AIDS ?
PCP
Which HIV pt. to give PCP
prophylaxis ?
All patients with a CD4 count <
200/mm³
Dx of PCP ?
✿ most common opportunistic infection in AIDS
✿ All patients with a CD4 count < 200 ➜ PCP prophylaxis
✿ Clinical pulmonary ➜ non-specific(Dyspnea,Dry cough,Fever,Very
few chest signs) Exercise-induced desaturation
✿ Extrapulmonary ➜ hepatosplenomegaly ,Lymphadenopathy ,Choroid
lesions
✿ CXR ➜ typically【 B/L INTERSTITIAL PULMONARY
INFILTRATES】 but can present with other x-ray findings e.g. lobar
consolidation. May be normal
✿ Sputum ➜ fails to Dx PCP
✿ BAL➜Dx PCP (SILVER STAIN)
Rx of PCP ?
✿ Co-trimoxazole
✿ IV pentamidine in severe cases
✿ Steroids if hypoxic (if pO2 <
9.3kpa)
MRCP Infectious Disease Revision notes
Dr.Sherif Elbadrawy
The most common cause of biliary
disease in patients with HIV is ?
【sclerosing cholangitis】 due to
infections such as CMV,
Cryptosporidium and Microsporidia
Pancreatitis é HIV infection dt ? anti-retroviral treatment
【didanosine】or by opportunistic
infections e.g. CMV.
Definition of Immune reconstitution
inflammatory syndrome (IRIS) ? ( ‫ﻋ‬‫ﺎ‬‫د‬
‫ﻟ‬‫ﯾ‬‫ﻧ‬‫ﺗ‬‫ﻘ‬‫م‬ )
when the immune system begins to recover (few
weeks after starting Anti-retroviral Rx)➜ responds
to a previously acquired opportunistic infection with
an overwhelming inflammatory response
➜paradoxically makes the symptoms of infection
worse.
✼【TB】 and cryptococcal meningitis.
Infections most commonly associated
with IRIS ?
【TB】 and cryptococcal meningitis.
Bacterial Meningitis CSF ?
✯ Cloudy
✯ Low Glucose (< 1/2 plasma)
✯ High Protein (> 1 g/l)
✯ 10 - 5,000 POLYMORPHS
MRCP Infectious Disease Revision notes
Dr.Sherif Elbadrawy
Viral Meningitis CSF ?
✯ Clear/cloudy
✯ Normal Glucose ➜ mumps & herpes
encephalitis ± low glucose
✯ Normal/raised Protein
✯ 15 - 1,000 LYMPHOCYTES
TB Meningitis CSF ?
✯ Fibrin web
✯ Low Glucose (< 1/2 plasma)
✯ High Protein (> 1 g/l)
✯ 15 - 1,000 LYMPHOCYTES
✯ PCR > better than Ziehl-Neelsen stain
Contraindications to LP ?
● Possible ↑ ICP
●Thrombocytopenia or other
bleeding diathesis (including
ongoing anticoagulant therapy)
●Suspected spinal epidural abscess
The most important α hemolytic
Streptococcus ?
● Streptococcus pneumoniae
(pneumococcus).
● a common cause of ➜ pneumonia,
meningitis and otitis media. Another
example is Strept viridans.
● Penicillin is the antibiotic of choice
The most important Group A β
hemolytic Streptococcus ?
● Streptococcus pyogenes
● erysipelas, impetigo, cellulitis, type 2
necrotizing fasciitis & pharyngitis/tonsillitis
● Immunological reactions➜ rheumatic fever
or post-streptococcal glomerulonephritis
● Erythrogenic toxins ➜ scarlet fever
● Penicillin is the antibiotic of choice
MRCP Infectious Disease Revision notes
Dr.Sherif Elbadrawy
The most important Group B β
hemolytic Streptococcus ?
Streptococcus agalactiae ➜ neonatal
meningitis and septicemia
CDC diagnostic criteria for
Staphylococcal toxic shock syndrome
?
✰ Fever: temperature > 38.9ºc
✰ Hypotension: SBP < 90 mmHg
✰ RASH Diffuse erythematous
✰ ±infected tampons
✰ Desquamation of rash, esp of the palms &
soles
✰ ≥ 3 organ systems
Types of Necrotising fasciitis
according to causative organism ?
✰ Type 1 is caused by mixed aerobes
& anaerobes (often occurs post-
surgery in diabetics)
✰ Type 2 is caused by Streptococcus
pyogenes
Clinical picture of Necrotising
fasciitis ?
✿ progressive destruction of fascia and SC
fat; ms is frequently spared dt ↑ blood
supply.
Acute ➜ skin color from red-purple to
patches of blue-gray ➜Extremely tender ➜
skin breakdown with bullae ➜ frank
gangrene
Rx of Necrotising fasciitis ? ✿ Urgent surgical debridement
✿ IV antibiotics
MRCP Infectious Disease Revision notes
Dr.Sherif Elbadrawy
Listeria monocytogenes mode of
infection ?
via CONTAMINATED
FOOD,unpasteurised dairy products
Vertical transmission➜ miscarriage
Clinical picture of Listeria infection ?
❂ CONTAMINATED FOOD,unpasteurised dairy products
➜【meningitis, meningoencephalitis, or bacteremia】in
IMMUNOSUPPRESSED PATIENTS, EXTREMES OF AGE (neonates
and elderly) and PREGNANT.
♞ diarrhoea, flu-like illness
♞ pneumonia , meningoencephalitis
♞ ataxia and seizures
❂ blood cultures. CSF may reveal a pleocytosis, with 'tumbling motility'
on wet mounts
❂ IV amoxicillin/ampicillin and gentamicin (resistant to
cephalosporins)
Dx of Listeria meningitis ? blood cultures. CSF may reveal a
pleocytosis, with 'tumbling motility'
on wet mounts
Rx of Listeria meningitis ?
Listeria meningitis should be treated
with IV amoxicillin/ampicillin and
gentamicin (resistant to
cephalosporins)
Clinical picture of Anthrax ?
❂ 【painless black eschar】 (cutaneous
'malignant pustule', but no pus)
❂ May cause marked edema
❂ Anthrax can cause gastrointestinal
bleeding
✿ ciprofloxacin
MRCP Infectious Disease Revision notes
Dr.Sherif Elbadrawy
Rx of Anthrax ? ✿ ciprofloxacin
Who should be screened for MRSA?
✿ All elective admissions except (day
patients➜ terminations
of pregnancy & ophthalmic surgery.
Patients admitted to mental health trusts)
✿ all emergency admissions
How should a patient be screened for
MRSA?
Nasal swab and skin lesions or
wounds
Treatment of invasive MRSA
infections in adults ?
✿ VANCOMYCIN
✿ DAPTOMYCIN
✿ TEICOPLANIN
✿ TIGECYCLINE
✿ QUINUPRISTIN & DALFOPRISTIN
✿ TELAVANCIN a semisynthetic lipoglycopeptide (-) cell wall
synthesis
✿ ARBEKACIN a semisynthetic aminoglycoside antibiotic
✿ CEFTAROLINE, a fifth generation cephalosporin
✿ Adjunctive agents → Rifampin & Fusidic acid
Etiology of Legionnaire's disease ?
✿ Legionella pneumophilia (intracellular
bacteria)
✿ colonizes water tanks and hence
questions may hint at air-conditioning
systems or foreign holidays. Person-to-
person transmission is not seen
MRCP Infectious Disease Revision notes
Dr.Sherif Elbadrawy
Clinical picture of Legionnaire's
disease ?
✿ Flu-like symptoms
✿ Dry cough
✿ Lymphopenia
✿ SIADH➜ Hyponatremia
✿ Deranged LFTs
✿ Urinary antigen
✿ Erythromycin
Dx of Legionnaire's disease ? Urinary antigen
Rx of Legionnaire's disease ? Erythromycin
Etiology of Leptospirosis (Weil's
disease) ?
❂ spirochaete Leptospira ➜spread by
contact with infected rat urine.( ‫ا‬‫ﻟ‬‫ﻔ‬‫ﺎ‬‫ر‬ ‫ﺑ‬‫و‬‫ل‬ )
❂ questions referring to sewage workers,
farmers, vets or people who work in abattoir.
Clinical picture of Leptospirosis ?
✿ Fever
✿ Flu-like symptoms ,no productive cough
✿ Jaundice
✿ (hepatorenal syndrome)(seen in 50% of patients)
✿ Weil's disease = Jaundice + Renal failure.
✿ Subconjunctival hemorrhage
✿ ±meningitis
❏ LP Dx meningitis
❏ High-dose 【benzylpenicillin or doxycycline】
MRCP Infectious Disease Revision notes
Dr.Sherif Elbadrawy
Rx of Leptospirosis ? ❏ Do LP 1st to Dx meningitis
❏ High-dose benzylpenicillin or
doxycycline
Etiology of Acute epiglottitis ? Hemophilus influenzae type B.
Clinical picture of Acute epiglottitis ?
❊ Hemophilus influenzae type B.
❊ children bw 2 and 6 years.
✿ Rapid onset
✿ Unwell, toxic child
✿ Stridor
✿ Drooling of saliva
✿ Rapid Dx & Rx is essential to prevent airway
obstruction
Etiology of Lyme Disease ? ✰ Borrelia burgdorferi (USA)
✰ Borrelia afzelii (EUROPE)
✰ Borrelia garinii(EUROPE)
Clinical picture of Lyme Disease ?
【Early features】
☆ ERYTHEMA CHRONICUM MIGRANS (small papule often at
site of the tick bite which develops into a larger annular lesion with
central
clearing, occurs in 70% of patients)
☆ Systemic symptoms: malaise, fever, arthralgia
【Later features】
☆ CVS: heart block, myocarditis
☆ Neurological: cranial nerve palsies, meningitis
☆ Polyarthritis
MRCP Infectious Disease Revision notes
Dr.Sherif Elbadrawy
Rx of Lyme Disease ?
☆【 Doxycycline】 (Should not be used for children <
8 years or pregnant or lactating women).
☆ ceftriaxone if disseminated disease
☆ Jarisch-Herxheimer reaction sometimes seen :
fever, rash, tachycardia after first dose of antibiotic
(> seen in syphilis, another spirochaetal disease) ➜
spontaneously settle after around four hours
screening for TB ? Heaf test & Mantoux test (more
accurate) ➜ to see if BCG is needed
Interpretation of Mantoux test ? Erythema & induration > 10mm =
positive result ➜ previous exposure
including BCG
Interpretation of Mantoux test ?
(details)
False negative Heaf & Mantoux tests
in ?
✪ Miliary TB
✪ Sarcoidosis
✪ HIV
✪ Lymphoma
✪ Very young age (e.g. < 6 months)
MRCP Infectious Disease Revision notes
Dr.Sherif Elbadrawy
Rx of active tuberculosis ?
❑ Initial phase - first 2 months (RIPE)
❁ Rifampicin
❁ Isoniazid
❁ Pyrazinamide
❁ Ethambutol
❑ Continuation phase - next 4 months
❁ Rifampicin
❁ Isoniazid
Rx of Latent tuberculosis ? isoniazid alone for 6 months
Meningeal tuberculosis ? prolonged period (at least 12 months)
with the addition of steroids
What's Directly observed therapy for
TB ?
❑ 3 per week dosing regimen may be
indicated in certain groups, including:
❁ Homeless people with active tuberculosis
❁ Patients who are likely to have poor
concordance
❁ All prisoners with active or latent
tuberculosis
SEs of Anti-TB medications ?
☠ All of them cause HEPATOTOXICITY 〖 + 〗
❏ Rifampin ➜ enzyme inducer [Revs up] , Red urine
❏ Isoniazid ➜ PERIPHERAL NEUROPATHY➜ Rx by
PYRIDOXINE, Enzyme INHibitor [INH = INHibits]
❏ Pyrazinamide ➜HYPERURICEMIA
❏ Ethambutol ➜ EYE ➜Optic neuritis/color vision
MRCP Infectious Disease Revision notes
Dr.Sherif Elbadrawy
Use of Streptomycin in TB ? aminogycoside used in resistant TB
Rx of Asymptomatic patient with Hx
of TB exposure and Heaf Test
positive ?
dual therapy (RI) for 5 months or
INH for 6 months.
Etiology of Leprosy ? Mycobacterium leprae
Mycobacterium lepromatosis.
Clinical picture of Leprosy ?
Initially asymptomatic for 5 to 20
years. ➜ granulomas of the nerves,
respiratory tract, skin (Nodular skin
lesions), and eyes, hypopigmented
skin lesions with reduced sensation.
Dx of Leprosy ?
Skin biopsy and needle test in cold area (ear
lobule & elbow) ➜ Lepromatous leprosy
➜slowly growing.【ACID ALCOHOL FAST
BACELLI】
tuberculoid leprosy ➜strong T cell immune
response with no bacelli in the affected area
skin biopsy.
MRCP Infectious Disease Revision notes
Dr.Sherif Elbadrawy
Rx of Leprosy ?
☯ Pauci-bacillary leprosy (<5 lesions)
rifampicin & dapsone for 6 months
☯ Multi-bacillary leprosy (>5 lesions)
rifampicin, dapsone & clofazimine for 12
months
Feature of severe malaria ? STAPH +
Complications
▼ S➜ Schizonts on a blood film
▼ T➜ Temperature > 39 °c
▼ A➜ Anemia (severe)
▼ P➜ Parasitemia > 2%
▼ H➜ Hypoglycemia
▼ + ANY Complication
Complications of malaria ?
▼ Brain➜ seizures, coma
▼ Lungs➜ ARDS
▼ Kidney➜ ARF: blackwater fever dt
intravascular
hemolysis
▼ Blood➜ DIC
▼ Pancreas➜ Hypoglycemia
Rx of Uncomplicated falciparum
malaria ?
✫ Strains resistant to chloroquine are prevalent in
Asia and Africa
✫ artemisinin ➜ first-line therapy
❐ artemether plus lumefantrine,
❐ artesunate plus amodiaquine, or mefloquine,or
sulfadoxine-pyrimethamine,
❐ dihydroartemisinin plus piperaquine
❐ Pregnant + malaria ➜ Quinine
Rx of Severe falciparum malaria ?
❂ A parasite counts >2% IV Rx 【IV
ARTESUNATE】
❂ If parasite count > 10% ➜ exchange
transfusion
❂ Shock may indicate coexistent bacterial
septicemia - malaria rarely causes
hemodynamic collapse
MRCP Infectious Disease Revision notes
Dr.Sherif Elbadrawy
the most common cause of Non-
falciparum malaria ?
【Plasmodium vivax】 ➜ Central
America & India
other causes ➜Plasmodium ovale &
Plasmodium malariae➜Africa
Rx of Non-falciparum malaria ? almost always chloroquine sensitive
destroy liver hypnozoites ➜ by
Primaquine (vivax & ovale)
prophylaxis of malaria ?
⓵ Atovaquone + proguanil (Malarone) ➜ GI upset
⓶ Chloroquine ➜ CI in epilepsy
⓷ Doxycycline ➜ Photosensitivity, Oesophagitis
⓸ Mefloquine ➜ Dizziness, Neuropsychiatric
disturbance, CI in epilepsy
✪ Pregnant ➜ chloroquine, proguanil + folate,
Atovaquone + proguanil (Malarone) + folate
✪ children over 2 months ➜ diethyltoluamide (DEET)
✪ children over 12 years ➜ doxycycline
Etiology of Leishmaniasis ?
intracellular protozoa
LEISHMANIA➜to humans by sand
flies➜incubation period up to 10
years.
Types of Leishmaniasis ? ✫ Cutaneous leishmaniasis
✫ Mucocutaneous leishmaniasis
✫ Visceral leishmaniasis (kala-azar)
MRCP Infectious Disease Revision notes
Dr.Sherif Elbadrawy
Clinical picture of Cutaneous
leishmaniasis ?
✫ Leishmania tropica or Leishmania
mexicana
✫ Crusted lesion at site of bite➜
ulcer
Clinical picture of Mucocutaneous
leishmaniasis ?
✫ Leishmania brasiliensis
✫ Skin lesions➜ spread to involve
mucosae of nose, pharynx
Clinical picture of Visceral
leishmaniasis (kala-azar) ?
✫ Leishmania donovani
✫ Mediterranean, Asia, South America, Africa
✫ Massive splenomegaly, hepatomegaly➜
Pancytopenia
✫ ↑ Fever,Typically ↓ appetite➜ weight loss (may be
paradoxical ↑ appetite + weight loss )
Types of Trypanosomiasis ?
African trypanosomiasis (sleeping
sickness)
American trypanosomiasis (Chagas'
disease)
Clinical picture of African
trypanosomiasis(sleeping sickness)?
✱ Trypanosoma gambiense in West Africa
✱ Trypanosoma rhodesiense in East Africa.
➜Both spread by 【tsetse fly】.
❂ rhodesiense ➜acute course➜Trypanosoma
chancre - tender SC nodule ➜post Cx
lymphadenopathy➜ Meningoencephalitis
MRCP Infectious Disease Revision notes
Dr.Sherif Elbadrawy
Rx of African
trypanosomiasis(sleeping sickness)?
Early disease: IV pentamidine or
suramin
Later disease or CNS involvement: IV
melarsoprol
Clinical picture of American
trypanosomiasis (Chagas' disease) ?
✱ Trypanosoma cruzi.
❁ Acute Chagas' disease ➜95% asymptomatic ±
CHAGOMA (an erythematous nodule at site of infection)
+ periorbital edema
❁ Chronic Chagas' disease:-
❅ HEART➜ Myocarditis ➜ HF & arrhythmias ➜ (leading
cause of death)
❅ GIT ➜ megaesophagus ➜dysphagia & megacolon ➜
constipation.
Rx of American trypanosomiasis
(Chagas' disease) ?
❍ Benznidazole or nifurtimox
❍ Chronic Rx the complications e.g.
heart failure
Etiology of Schistosomiasis ?
♕ Schistosoma Hematobium➜ urinary
schistosomiasis
【hematobium = hematuria】
Schistosomiasis is the most common cause of bladder
calcification worldwide
♕ Schistosoma mansoni➜ intestinal schistosomiasis
NB: Schistosoma mansoni & japonicum ➜ mesentric
veins is the final distenation ➜ GIT symptoms.
Clinical picture of Schistosoma
Hematobium ?
a 'swimmer's itch' in patients who have recently
returned from Africa.
Schistosoma Hematobium➜Frequency,Hematuria
&Bladder calcification.
a risk factor for squamous cell bladder cancer
Single oral dose of 【PRAZIQUANTEL】
MRCP Infectious Disease Revision notes
Dr.Sherif Elbadrawy
Rx of Schistosoma Hematobium ? Single oral dose of
【PRAZIQUANTEL】
Clinical picture of Rabies ?
Prodrome: headache, fever, agitation
HYDROPHOBIA: water-provoking
muscle spasms
Hypersalivation
What to do after an animal bite in at
risk countries ?
❊ If immunised➜ 2 doses of vaccine
❊ If not immunised ➜ (HRIG) + full
vaccination
❊ Cleanse wound
❊ CO-AMOXICLAV (DOCH)
❊ If penicillin-allergic ➜ doxycycline +
metronidazole is recommended
Etiology of Cat scratch disease ? Bartonella henselae Gram negative
rod
Clinical picture of Cat scratch disease
?
❍ Bartonella henselae Gram negative
rod
❍ fever ,headache, malaise
❍ regional lymphadenopathy
MRCP Infectious Disease Revision notes
Dr.Sherif Elbadrawy
Etiology of Chickenpox ? primary infection with VZV
shingles is reactivation of dormant
virus in dorsal root ganglion
MOST CONTAGIOUS ORGANISM ? VZV
Mode of transmission of VZV ?
✱ respiratory route
✱ Through shingles
✱ Infectivity = 4 days before rash, until 5
days after the rash(till all lesions had
scabbed over)
✱ Incubation period = 11-21 days
Clinical picture of VZV ?
✬ Fever initially
✬ Itchy, rash starting on head/trunk
➜ spread.1st macular then papular
then vesicular
✬ Systemic upset is usually mild
Rx of VZV ?
❂ Keep cool, trim nails
❂ Calamine lotion
❂ School exclusion:5 days from start of rash,
until all lesions are crusted & dry.
❂ VZIG (for Immunocompromised patients
and newborns with peripartum exposure)➜
If chickenpox develops ➜ IV aciclovir
MRCP Infectious Disease Revision notes
Dr.Sherif Elbadrawy
complications of VZV ?
❂ Secondary bacterial infection of the
lesions
❂ Pneumonia➜ MC & serious ➜ IV
acyclovir
❂ Encephalitis ± cerebellar
❂ Disseminated hemorrhagic chickenpox
Dx of VZV infection In pregnancy ?
❂ first step is to check antibodies
❂ both mother & fetus can be affected, a
syndrome called fetal varicella syndrome
✺ not immune ➜ VZIG ASAP
✺ oral aciclovir if pregnant women with
chickenpox present within 24 h of rash
Clinical picture of FVS {Fetal
varicella syndrome} ?
✯ before 20 weeks
✯ skin scarring,microphthalmia,
microcephaly & learning disabilities,
limb hypoplasia,
Rx of VZV infection In pregnancy ? ✺ not immune to varicella ➜ VZIG ASAP
✺ oral aciclovir if pregnant women with
chickenpox present within 24 h of rash
Etiology of Measles ?
✺ RNA paramyxovirus
✺ Spread by droplets
✺ Infective from prodrome until 5
days after rash starts
MRCP Infectious Disease Revision notes
Dr.Sherif Elbadrawy
Clinical picture of Measles ?
✺ Prodrome: irritable, conjunctivitis, fever
✺ Koplik spots (before rash): white spots
('grain of salt') on buccal mucosa
✺ Rash: starts behind ears then to whole
body, discrete maculopapular rash becoming
blotchy & confluent
Complications of Measles ?
[NB:Pancreatitis and infertility may
follow mumps infection]
✺ Encephalitis: 1-2 weeks after the onset of the illness.
✺ Febrile convulsions
✺ Pneumonia, tracheitis
✺ Keratoconjunctivitis, corneal ulceration
✺ Diarrhoea
✺ ↑ incidence of appendicitis
✺ Myocarditis
✺ Subacute sclerosing panencephalitis: very rare, may
present 5-10 years following the illness
Management of contacts of Measles ?
✺ a child not immunized + contact
with measles then MMR (vaccine-
induced measles antibody > rapid
than natural infection)
✺ Use within 72 hours
Etiology of Gonorrhoea ?
Gram negative diplococcus Neisseria
gonorrhoea
Acute infection can occur on any MM
surface, typically genitourinary but
also rectum and pharynx
Clinical picture of Gonorrhoea ?
✺ ♂ urethral discharge, dysuria
✺ ♀ cervicitis ➜ vaginal discharge
✺ Rectal and pharyngeal infection is usually
asymptomatic, but may present as rectal bleeding
✺ Local complications➜ urethral strictures,
epididymitis and salpingitis ➜ (infertility).
Disseminated infection may occur
MRCP Infectious Disease Revision notes
Dr.Sherif Elbadrawy
Rx of Gonorrhoea ?
✺ Cephalosporins ("Oral" Cefixime or
Ceftriaxone "single dose IM") ➜ DOC
✺ Ciprofloxacin is no more DOC dt ↑
resistance to ciprofloxacin
Etiology of Disseminated gonococcal
infection ?
✩ Hematogenous spread from asymptomatic genital
infection.
✩ Triad : 【tenosynovitis, migratory polyarthritis
and dermatitis】 (maculopapular or vesicular)
✩ Later complications ➜ septic arthritis(MCC of
septic arthritis in young adults), endocarditis and
perihepatitis
Etiology of Genital warts ?
(condylomata accuminata)
human papilloma virus HPV
especially types 6 & 11
Which types of human papilloma
virus predispose to cervical cancer ?
(16, 18 & 33)
Clinical picture of Genital warts ?
(condylomata accuminata)
HPV, Small (2 - 5 mm) fleshy
protuberances on the genital regions
+ slightly pigmented
bleeding or itching.
MRCP Infectious Disease Revision notes
Dr.Sherif Elbadrawy
Rx of Genital warts ? (condylomata
accuminata)
★ Topical podophyllum or cryotherapy➜ first-line
★ Multiple, non-keratinised warts ➜ Topical
podophyllum
★ solitary keratinised warts ➜ cryotherapy,
Imiquimod
★ recurrence is common
★ HPV clear without intervention within 1-2 years
Etiology of Genital Ulcers ?
Syphilis➜Painless
Lymphogranuloma venereum➜Painless
Granuloma inguinale➜Painless
Herpes Simplex Virus➜Painful
Chancroid➜Painful
Dx of Syphilis ?
Treponema pallidum is a very sensitive
organism can't grow on artificial media.
The diagnosis is therefore usually based
on clinical features; serological tests and
microscopic examination of infected
tissue
Serological tests for Dx of Syphilis ?
✾ Cardiolipin tests (not treponeme specific)
✷ VDRL & RPR
✷ Insensitive in late syphilis
✷ Becomes negative after treatment
✾ Treponemal specific antibody tests
✷ TPHA
✷ Remains positive after treatment
Causes of false positive cardiolipin
tests
❂ pregnancy
❂ SLE, anti-phospholipid syndrome
❂ TB
❂ leprosy
❂ malaria
❂ HIV
MRCP Infectious Disease Revision notes
Dr.Sherif Elbadrawy
Etiology of Lymphogranuloma
venereum ?
Chlamydia trachomatis.
Clinical picture of Lymphogranuloma
venereum ?
Chlamydia trachomatis.
✾ Stage 1: painless pustule ➜ ulcerate
✾ Stage 2: painful inguinal
lymphadenopathy
✾ Stage 3: proctocolitis
Types of Herpes Simplex Virus ?
HSV-1 and HSV-2
it was thought HSV-1 ➜ oral lesions (cold
sores) & HSV-2 ➜genital herpes it is now
known there is considerable overlap
Clinical picture of Herpes Simplex
Virus ?
❂ Cold sores
❂ Painful genital ulceration
❂ Primary infection ➜ ±
gingivostomatitis
Rx of Herpes Simplex Virus ?
❂ Gingivostomatitis: oral aciclovir,
chlorhexidine MW
❂ Cold sores: topical aciclovir
❂ Genital herpes: oral aciclovir,
↑exacerbations ➜ longer term aciclovir
MRCP Infectious Disease Revision notes
Dr.Sherif Elbadrawy
Etiology of Chancroid ? Hemophilus ducreyi
Clinical picture of Chancroid ?
❃ Hemophilus ducreyi
❃ painful genital ulcers
❃ inguinal lymphadenopathy.
❃ Grey or yellow purulent exudate⇔ Syphilis
is Non-exudative
❃ Soft edge ⇔ Syphilis is Hard (indurated)
edge
Common causes of Vaginal discharge
?
✯ Physiological
✯ Candida➜'Cottage cheese' discharge Vulvitis Itch
✯ Trichomonas vaginalis➜ Offensive, yellow/green,
frothy discharge, Vulvovaginitis Strawberry cervix
✯ Bacterial vaginosis ➜Offensive, thin, white/grey,
'FISHY' discharge
Less common causes of Vaginal
discharge ?
✯ cervical infections such as Chlamydia
and Gonorrhoea ➜rarely the presenting
symptoms
✯ Ectropion
✯ Foreign body
✯ Cervical cancer
Etiology of Bacterial vaginosis (BV) ?
✯ overgrowth of anaerobic organisms➜
【Gardnerella vaginalis】 ➜ ↓ lactobacilli
➜↓ Lactic acid ➜↑ vaginal pH. not a STD
but seen almost exclusively in sexually active
women.
✯ Asymptomatic in 50%
MRCP Infectious Disease Revision notes
Dr.Sherif Elbadrawy
Amsel's criteria for diagnosis of
Bacterial vaginosis (BV) ?
【Gardnerella vaginalis】
❃ 3 of the following 4 points
✴ Thin, white homogenous discharge
✴ Clue cells on microscopy
✴ Vaginal pH > 4.5
✴ Positive whiff test (addition of
potassium hydroxide ➜ fishy odour)
Complications of Bacterial vaginosis
in pregnancy ?
✯ preterm labour
✯ low birth weight
✯ chorioamnionitis
✯ late miscarriage
✯ oral metronidazole is used throughout
pregnancy.
Rx of Bacterial vaginosis (BV) ?
✯ Oral metronidazole for 5-7 days
✯ 70-80% initial cure rate
✯ Relapse rate > 50% within 3
months
most common sexually transmitted
infection in the UK ?
✯ Chlamydia
✯ caused by Chlamydia trachomatis,
an obligate intracellular pathogen
✯ ≅ 10 % in UK women.
Clinical picture of Genital Chlamydia
infection ?
Asymptomatic in around 70% of
women
Women: cervicitis (discharge,
bleeding), dysuria
Men: urethral discharge, dysuria
MRCP Infectious Disease Revision notes
Dr.Sherif Elbadrawy
complications of Genital Chlamydia
infection ?
✯ Epididymitis
✯ Pelvic inflammatory disease
✯ Endometritis ➜ 【1st 3 causes➜
Infertility】
✯ ↑ incidence of ectopic pregnancies
✯ Reactive arthritis
✯ Perihepatitis (Fitz-Hugh-Curtis syndrome)
Dx of Genital Chlamydia infection ?
✯ no Traditional cell culture
✯ Ix of choice➜Nuclear Acid Amplification Tests
(NAATs)➜Use Urine (first void urine sample),
vulvovaginal swab or cervical swab
✯ NB: Neisseria Gonorrhoea ➜ Gram stain➜ Gram -
ve diplococci ➜ If the swab showed non-specific
urethritis ➜ Chlamydia is most likely. Both many
times infect together
Screening of Genital Chlamydia
infection ?
screening all sexually active patients
aged 15-24 years
Rx of Genital Chlamydia infection ?
✿ 【 Azithromycin 】〖first-line〗➜single dose➜better compliance
✿ Doxycycline 7 day course ➜may be used but Azithromycin is better dt
better compliance.
✿ pregnant➜ erythromycin or amoxicillin
✿ in ♂ with symptomatic infection partner notification➜ (all partners
from 4 weeks prior to symptoms)
✿ in ♀& asymptomatic ♂ partner notification ➜ ( last 6 months or the
most recent sexual partner)
✿ partners of confirmed Chlamydia Pt ➜treat then test
✿ A test of cure following treatment.
Clinical picture of Chlamydia psittaci
(psittacosis)(parrot disease, parrot
fever) ?
malaise, fever, myalgias and
pneumonia
Exposure to an ill bird and a rash
(Horder's spots) are pathognomonic.
MRCP Infectious Disease Revision notes
Dr.Sherif Elbadrawy
Rx of Chlamydia psittaci (psittacosis)
?
Erythromycin or tetracyclines are the
drugs of choice.
Rx of Asymptomatic bacteruria in
pregnant women ?
amoxicillin or cephalosporin
Rx of Lower urinary tract infections
in women (cystitis) ?
trimethoprim or nitrofurantoin for 3
days
Rx of acute lower urinary tract
infections in pregnant women ?
amoxicillin or an oral cephalosporin
for 7 days
Rx of acute pyelonephritis ?
✼ hospital admission
✼ a broad-spectrum cephalosporin or a
quinolone for 10-14 days
✼ ciprofloxacin for 7 days or co-
amoxiclav for 14 days
MRCP Infectious Disease Revision notes
Dr.Sherif Elbadrawy
most common congenital infection in
the UK ?
Cytomegalovirus 〚Maternal
infection is usually asymptomatic〛
congenital Rubella infection ?
PDA = (P)atent DA, (D)eafness,
(A)nterior chamber
【CATARACTS & Glaucoma】 ➜
Anterior chamber
【heart disease (PDA)】
【Sensorineural deafness】
congenital Toxoplasma infection ?
【CEREBRAL CALCIFICATION➜
Hydrocephalus】
【CHORIORETINITIS】 ➜ Posterior
chamber
congenital CMV infection ? 【Growth retardation】
【Purpuric skin lesions】
Less diagnostic features of congenital
Toxoplasmosis, Rubella & CMV ?
〖All of them〗➜ Hepatosplenomegaly,Cerebral
palsy
【Toxoplasmosis】➜ Anemia
【Rubella】➜ Purpuric skin lesions 'Salt and pepper'
,chorioretinitis
Microphthalmia ,Growth retardation
【CMV】➜ Anemia, Sensorineural deafness,
Encephalitiis ,Pneumonitis, Jaundice
MRCP Infectious Disease Revision notes
Dr.Sherif Elbadrawy
Etiology of Toxoplasmosis ?
Toxoplasma Gondii➜ a protozoa➜ infection via GIT, lung or
broken skin➜the oocysts release trophozoites ➜migrate widely
around the body including to the eye, brain and muscle.
The usual animal reservoir is the CAT
± rats may carry the disease.
Clinical picture of Toxoplasmosis ?
✾ mostly asymptomatic
✾ If symptomatic➜ self-limiting infection
✾ same as infectious mononucleosis
(fever, malaise, and lymphadenopathy).
✾ meningioencephalitis and myocarditis.
(< common)
Dx of Toxoplasmosis ? Antibody test
Sabin-Feldman dye test
Rx of Toxoplasmosis ? Pyrimethamine + Sulphadiazine for
at least 6 weeks.
Mnemonic for Infectious Causes of
Bloody Diarrhea ?
CHESS
Campylobacter
Hemorrhagic E. coli (O157:H7)
Entamoeba histolytica
Salmonella
Shigella
MRCP Infectious Disease Revision notes
Dr.Sherif Elbadrawy
The most common cause of
Travellers' diarrhea ?
Escherichia coli
at least 3 loose to watery stools in 24
h
± Abdominal pain, fever, nausea,
vomiting or blood in the stool
most common infection from
reheated rice ?
Bacillus cereus infection
Etiology of 'acute food poisoning' ?
Staphylococcus aureus,
Bacillus cereus
Clostridium perfringens.
sudden onset of nausea, vomiting and
diarrhea after the ingestion of a toxin.
Clinical picture of Cholera diarrhea ?
Vibro cholerae - Gram negative bacteria
Profuse rice watery diarrhea
Severe dehydration ➜ weight loss
Not common amongst travellers
Clinical picture of Campylobacter
diarrhea ?
✯ A flu-like prodrome ➜ abdominal pains➜
Fever and diarrhoea ± blood
✯ may be complicated by Guillain-Barre
syndrome
MRCP Infectious Disease Revision notes
Dr.Sherif Elbadrawy
Campylobacter Rx ?
✯ usually self-limiting
✯ immunocompromised ➜ antibiotics if
severe symptoms [bloody diarrhoea, > 8
stools / day] ➜ clarithromycin
Clinical picture of Amoebiasis
diarrhea ?
Gradual onset bloody diarrhea
Abdominal pain and tenderness
May last for several weeks
Differnt Incubation periods of
Diarrhea ?
✯ 1-6 hrs: Staphylococcus aureus,
Bacillus cereus
✯ 12-48 hrs: Salmonella, Escherichia coli
✯ 48-72 hrs: Shigella, Campylobacter
✯ > 7 days: Giardiasis, Amoebiasis
Rx of Cholera diarrhea ? ✯ Oral rehydration therapy
✯ doxycycline, ciprofloxacin
Etiology of Giardiasis ? flagellate protozoan Giardia lamblia.
It is spread by the faeco-oral route
MRCP Infectious Disease Revision notes
Dr.Sherif Elbadrawy
Clinical picture of Giardiasis ?
✯ Often asymptomatic
✯ Lethargy, bloating, abdominal pain
✯ Non-bloody diarrhoea
✯ Chronic/prolonged diarrhoea,
malabsorption and lactose intolerance
Dx of Giardiasis ?
Stool microscopy for trophozoite and
cysts are CLASSICALLY NEGATIVE
➜ duodenal fluid aspirates or 'string
tests' (fluid absorbed onto swallowed
string) are sometimes needed.
Rx of Giardiasis ? Metronidazole
Loa Loa CP
▩ CP ➜ pruritus, urticaria, Calabar swellings: transient, swelling
of soft-tissue around joints, 'eye worm' - the dramatic presentation
of subconjuctival migration of the adult worm.
♕ Rx ➜ diethylcarbamazine (DEC),
high loa loa microfilaraemia is associated with encephalopathy
following Rx with either Ivermectin or DEC. dt death of vast
numbers of blood microfilaria.
Both of these drugs are CI if loa loa microfilaraemia > 2500 mf/ml.
Most common E -coli infections in
humans ?
✯ Diarrhoeal illnesse
✯ UTIs
✯ Neonatal meningitis
MRCP Infectious Disease Revision notes
Dr.Sherif Elbadrawy
Clinical picture of EHEC (E coli
O157:H7) infection ?
✯ Diahrrea
✯ Renal Failure or Impairment
✯ Thrombocytopenia
✯ hemolytic uremic syndrome
✯ hemorrhage➜ ↓ Hb
hemolytic uremic syndrome can be
caused by which bacteria ?
EHEC (E coli O157:H7) infection
spread by contaminated ground beef.
Which disease Can Body louse
transmit ?
✯ Body lice can spread epidemic typhus, trench fever, and louse-
borne relapsing fever.
✯ louse-borne (epidemic) typhus is no longer widespread,
✯ outbreaks of this disease still occur during overcrowedness with
low sanitation:-
◌ times of war,
◌ civil unrest,
◌ natural or man-made disasters,
◌ in prisons
◌ chronic poverty
Clinical picture of Salmonella
infection (Typhoid) ?
❂ headache,fever, and arthralgia
❂ Relative bradycardia
❂ Abdominal pain, distension
❂ Constipation: > in typhoid
❂ Rose spots: > in paratyphoid
complications of Salmonella
infection (Typhoid) ?
❂ Osteomyelitis (Salmonella is MCC in
sickle cell disease Osteomyelitis )
❂ GI bleed/perforation
❂ Meningitis
❂ chronic carriage (1%, more likely if
adult females)
MRCP Infectious Disease Revision notes
Dr.Sherif Elbadrawy
Types of Shigella bacterial diarrhea ?
S sonnei (e.g. from UK) may be mild,
S flexneri or S dysenteriae
from abroad may cause severe
disease
Rx of Shigella bacterial diarrhea ? Ciprofloxacin
Definition of Pyrexia of Unkown
Origin ?
a fever of > 3 weeks which
undiagnosed after a week in hospital
Etiology of Pyrexia of Unkown Origin
?《 Infections Can Make
Undergraduates Die》 ?
〖Infection (20-40%)〗
❂ TB
❂ endocarditis
❂ EBV
❂ CMV
〖Connective tissue disorders (20%)〗
❂ RA
❂ SLE
❂ Adult-onset Still's disease
❂ Temporal arteritis
〖Malignancy (10-20%)〗
❂ Lymphoma
❂ Leukaemia
❂ HCC
❂ RCC
❂ Atrial myxoma
〖Undiagnosed (20%)〗
〖Drugs〗phenytoin
Clinical picture of African Tick
Typhus ?
dt Rickettsiae conorii
BLACK SPOTS ON THIGH
Hx of tick bites
Low grade fever
Faint macular rash
MRCP Infectious Disease Revision notes
Dr.Sherif Elbadrawy
Etiology of Rocky Mountain Spotted
Fever ?
Rickettsia,spreads by ticks, common
in USA.
Clinical picture of Rocky Mountain
Spotted Fever ?
❀ Rickettsia,spreads by ticks, common in
USA.
❀ Fever
❀ Rash on hands, feet ➜ peel
❀ Tachycardia with no hypotension (unlike
Staphy Toxic Shock Syndrome)
❀ Rx Doxycycline
Rx of Rocky Mountain Spotted Fever
?(Rickettsia)
Doxycycline
Etiology of Mediterranean Spotted
Fever ?(Boutonneuse fever)
Rickettsia conorii and transmitted by
the dog tick Rhipicephalus
sanguineus
Clinical picture of Mediterranean
Spotted Fever ?(Boutonneuse fever)
↑ FEVERS, MYALGIA and joints pain, severe
headache, photophobia and diarrhea.
bite site ➜ black spots or ulcerous crust
(tache noire).4th day ➜
widespread rash appears, first macular ➜
maculopapular ± petechial.
MRCP Infectious Disease Revision notes
Dr.Sherif Elbadrawy
Rx of Mediterranean Spotted Fever ? Doxycycline.
Etiology of viral hemorrhagic fever ?
❂ Dengue fever
❂ yellow fever
❂ Lassa fever
❂ Ebola
Given the current outbreak in West Africa, this group
of infections must be considered as a differential in
returning travellers presenting with a fever.
Etiology of Dengue fever ? dengue virus Transmitted by the
Aedes aegypti mosquito
Clinical picture of Dengue fever ?
❀ Dengue virus (Aedes aegypti mosquito)
❀ incubation period of 7 days
❀ Headache (often retro-orbital)
❀ Facial flushing (dengue)
❀ Maculopapular rash (resembling measles)
❀ Thrombocytopenia &↑ LFTs
❀ DIC
❀ Rx ➜ symptomatic ➜ fluid resuscitation, blood
transfusion
Rx of Dengue fever ? symptomatic ➜fluid resuscitation,
blood transfusion
MRCP Infectious Disease Revision notes
Dr.Sherif Elbadrawy
Etiology of Infectious mononucleosis
(glandular fever) ?
❀ Epstein-Barr virus ( HHV-4).
❀ adolescents and young adults.
Clinical picture of Infectious
mononucleosis (glandular fever) ?
❀ Sore throat,Fever.Palatal petechiae
❀ Lymphadenopathy
❀ Splenomegaly in 50% of patients ➜ ± splenic
rupture
❀ Hepatitis
❀ Malaise, anorexia, headache
❀ lymphocytosis
❀ Hemolytic anaemia
Rx of Infectious mononucleosis
(glandular fever) ?
❀ symptomatic ➜fluid resuscitation,Rest
during the early stages,avoid alcohol
❀ analgesics
❀ avoid contact sports for 8 weeks
➜↓risk of splenic rupture
Malignancies associated with EBV
infection ?
Burkitt's lymphoma
Hodgkin's lymphoma
Nasopharyngeal carcinoma
Criteria Hepatitis E infection ?
RNA virus
faecal-oral route
Common in Asia, Africa & Mexico
significant mortality (about 20%) during
pregnancy
Does not cause chronic disease
no vaccine
MRCP Infectious Disease Revision notes
Dr.Sherif Elbadrawy
Etiology of Swine Flu ?
influenza A virus and the most
common cause of flu in humans. The
2009 pandemic was caused by a new
strain of the H1N1 virus.
Pts at risk of swine flu ?
Patients with chronic Disease + Pts
on immunosuppressants
Pregnant women
Young children < 5 years old
Clinical picture of swine flu ?
Fever > 38ºC
Myalgia
Lethargy
Headache
Rhinitis
Sore throat
Cough
Diarrhoea and vomiting
may be complicated by ARDS.
Rx of swine flu ? (H1N1) (ARDS)
❀ Oseltamivir (Tamiflu):-
Oral medication, A neuraminidase inhibitor
SEs ➜ NVD & headaches
❀ Zanamivir (Relenza)
Inhaled or IV for acutely ill pts.
A neuraminidase inhibitor
SEs ➜ bronchospasm in asthmatics
Etiology of Erythema infectiosum
(fifth disease or 'slapped-cheek
syndrome') ?
Parvovirus B19
MRCP Infectious Disease Revision notes
Dr.Sherif Elbadrawy
Clinical picture of Erythema
infectiosum (fifth disease or 'slapped-
cheek syndrome') ?
❂ Parvovirus B19
❂ may be Asymptomatic
❂ lethargy, fever, headache
❂ 'slapped-cheek' rash spreading to proximal
arms and extensor surfaces
❂ Pancytopenia in immunosuppressed
❂ Aplastic crises in sickle-cell disease
Etiology of Orf ?
parapox virus
a condition found in sheep and goats
although it can be transmitted to
humans.
Clinical picture of Orf ?
❂ parapox virus
❂ a condition found in sheep and goats
❂ small, raised,papules ( red-blue)in the
hands and arms ➜↑ in size,become flat-
topped and hemorrhagic
Most common cause of cutaneous
larva migrans ?
ancylostoma braziliense
Common in Central and Southern
America
Strongyloides stercoralis causes what
?
✾ larva currens - similar appearance to cutaneous larva
migrans but moves through the skin at a far greater rate
✾ Acquired percutaneously (e.g. Walking barefoot)
✾ EOSINOPHILIA
✾ pruritus
✾ Abdominal pain, diarrhoea, pneumonitis
✾ May cause gram negative septicemia due to carrying of
bacteria into bloodstream
MRCP Infectious Disease Revision notes
Dr.Sherif Elbadrawy
Rx of Strongyloides stercoralis ? thiabendazole, albendazole.
Ivermectin also used, particularly in
chronic infections.
Commonest cause of visceral larva
migrans ?
TOXOCARA CANIS
dt ingesting eggs from soil
contaminated by dog faeces
eye granulomas, liver/lung
involvement
Etiology of Cysticercosis ? Taenia solium (from pork) and
Taenia saginata (from beef).
Rx niclosamide
Rx of Cysticercosis ? niclosamide
Etiology of Hydatid disease ?
dog tapeworm Echinococcus
granulosus
Often seen in farmers
May cause liver cysts
Rx Albendazole
MRCP Infectious Disease Revision notes
Dr.Sherif Elbadrawy
Rx of Hydatid disease ? Albendazole
The most common infecting
organism after a dog bite ?
Anaerobic mouth flora pasteurella
multocida
capnocytophaga & strept pyogenes may be
present
Commonest cause of viral
gastroenteritis ?
✬ Norwalk virus (norovirus) ➜Watery
diarrhea
✬ sporadic & epidemic diarrhea
✬ Dx by ELISA ➜ viral Ag detection
✬ (other viral causes of watery diarrhea
include Rotavirus and Adenovirus)
The rapid development of focal
neurological signs, in the context of
severe immunosuppression with HIV and
the
presence of multiple ring-enhancing
lesions in the brain ?
✬ toxoplasmosis
Tuberculosis, progressive multifocal
leucoencephalopathy (hyperintense signal on
T2-weighted images,cerebellar), lymphoma a
(solitary lesion) & HIV encephalopathy are
also possible diagnoses in the present case.
Definition of Tropical sprue ?
a malabsorption disease commonly found
in the tropical regions, marked with
abnormal flattening of the villi and
inflammation of the lining of the small
intestine.
MRCP Infectious Disease Revision notes
Dr.Sherif Elbadrawy
pigment producing bacteria:
"ISRAEL has YELLOW SAND" ?
actinomyces israeli - yellow "sulfur"
granules
primary affect face & neck➜usually
present as abscess.
HIV-positive. Her CD4 count is > 200 and she
presented with salmonella gastroenteritis. Rx successfully.
In addition to (HAART), we should give prophylaxis
against which one of the following organisms ?
Toxoplasma gondii
Cytomegalovirus
Pneumocystis jiroveci
Salmonellae
Clyptococci
✬ SALMONELLAE
✬ prophylaxis for (P. jiroveci, cytomegalovirus, Toxopiasma and
cryptococcal infections) can be D/C in patients with CD4 > 200.
Long-term therapy ➜to prevent a recurrence in people diagnosed
with salmonellosis. ➜ despite HAART, patients infected with HIV
appear to mount an ineffective antibody response to salmonella
infection.
✬ Ciprofloxacin is usually the first choice
Which one of the following options is true with respect to
prophylaxis against pneumococcal infection in post splenectomy
pt.?
☆ Penicillin is not indicated
☆ Penicillin prophylaxis 500 mg bd is indicated for at least a 2-
year period
☆ Penicillin prophylaxis 250 mg bd is indicated for a 2-year period
☆ They should be immediately revaccinated against pneumococcus
☆ Pneumococcal vaccination should be repeated every 10 years
☆ Penicillin prophylaxis 500 mg bd
is indicated for at least a 2-year
period !!
☆ Pneumococcal vaccination should
be repeated every 5 years
risk of developing various infections
and oppertunistic diseases at
different CD4 counts landmarks in
HIV infected patients ?
☆ CD4 count above 700 ➜Normal
☆ CD4 count 200-500 ➜ Oral thrush, Kaposi sarcoma,
TB, Zoster, and lymphoma
☆ CD4 count 100-200 ➜ PCP pneumonia, PML,
Dementia, Histoplasmosis, and Coccidiomycosis
☆ CD4 count 50-100 ➜ Toxoplasmosis, Cryptococcosis,
and Cryptosporodiosis
☆ CD4 count less than 50 ➜ CMV, Mycobacterium avium
complex, CNS lymphoma
characteristic diseases in AIDS
patients ?
☆ Hairy Leukoplakia
☆ Kaposi Sarcoma (CD Count: 200 - 500 cells/ml)
☆ High-grade B-Cell Lymphomas
☆ Cervical cancer
☆ Wasting syndrome
☆ Nephropathy
☆ Dementia (CD Count: 100 - 200 cells/ml)
MRCP Infectious Disease Revision notes
Dr.Sherif Elbadrawy
AIDS Oppurtunistic Infections ?
✪ LUNG -
✷ PCP - pneumonia, also in bone marrow (CD100 - 200 )
✷ Mycobacterium TB - (CD : 200 - 500)
✷ Mycobacterium avium-intracellulare - also in GI tract (CD: <50)
✷ Coccidioidomycosis
✷ Histoplasmosis
✷ CMV - also affect retina, adrenals and GI tract (CD: <50)
✷ Cryptococcus
✪ Esophagus -
✷ HSV - also cause CNS encephalitis (CD : 200 - 500)
✷ Candida - oral thrush in oral pharynx (CD: 200 - 500)
✪ GI Tract -
✷ Giardia Lamblia
✷ Cryptosporidium
✪ CNS -
✷ Aspergillus - spores can also affect lungs and blood vessels
✷ Toxoplasmosis - MC opportunistic infection of brain (CD : <100)
✷ Cryptococcus -meningitis(most common presentation for cryptococcus "also in the lung") (chronic only, not acute) (CD
:<100 )
✷ JC virus - progressive multifocal leukoencephalopathy (CD: <50)
When should the patient in a pre splenectomy case
receive conjugate pneumococcal vaccination?
✬ 1 week before operation
✬ 5 days before operation
✬ 2 weeks before operation
✬ 1 month after operation
✬ Postoperatively
2 weeks before operation
also Hib & meningococal vaccine
A throat swab reveals diphtheria Which one of the
following options is the next most appropriate
action?
✬ Examine the cerebrospinal fluid (CSF)
✬ Blood cultures
✬ Ceftriaxone
✬ Antitoxin
✬ Hydrocortisone
Antitoxin
Clinical picture of Herpes simplex
virus kerariris ?
- Herpes simplex virus (HSV) keratitis is
characterised by the acute onset of pain,
blurred vision, conjunctival injection and
DENDRITIC ULCERATION of the cornea
- HSV keratitis can cause corneal blindness
and its treatment is urgent
Which one of the following options is the
most common side-effect of pyrazinamide?
Hepatitic dysfunction
Hyperuricaemia
Colour vision changes
Dizziness
Neurotoxicity
Hepatitic dysfunction
Hyperuricaemia is a characteristic SE
but not the most common.
MRCP Infectious Disease Revision notes
Dr.Sherif Elbadrawy
Clinical picture of chickenpox ?
highly contagious disease caused by primary infection
with varicella zoster virus (VZV)
prodrome of constitutional symptoms ➜ characteristic
vesicular rash all over the body and/or oral sores ➜ blister
stage, intense itching is usually present. Blisters may also
occur on the palms, soles, and genital area. ± oral cavity
& tonsil areas in the form of small ulcers which can be
painful or itchy or both
Can Interferon administration in the
acute phase of infection prevents the
development of the chronic hepatitis
B carrier state ?
【NO】
Interferon administration in the
acute phase of infection has not
shown any benifit.but it's effective in
the Rx of chronic hepatitis B.
Etiology of Yellow fever ?
flavivirus, and can vary in severity from a
mild illness to the severe classical form.
present in TROPICAL areas➜ Africa and
South America ➜ transmitted to human
by mosquito.
Clinical picture of Yellow fever ?
⌘ Aedes mosquitos
⌘ incubation period = 2 - 14 days
⌘ severe flu-like illness with fever up to 40 ± epigastric pain and
vomiting
⌘ Relative bradycardia
⌘ a recovery phase and the patient feels well for several days
⌘ severe fever again+ jaundiced, hepatomegaly, ➜ haematemesis,
bleeding gums ➜ DIC.
⌘ Councilman bodies (inclusion bodies) may be seen in the
hepatocytes
Rx of Yellow fever ? Supportive therapy only
Mortality up to 40%
MRCP Infectious Disease Revision notes
Dr.Sherif Elbadrawy
Dx of malaria ?
❂ Three separate blood films should be sent to r/o
malaria when it is suspected.
❂ Thick & thin film blood smear ➜ film is +ve ➜
send the other blood samples.. if the pt is severely ill
or deteriorating ➜ start antimalarial Rx ASAP before
C&S result.
❂ Although the patient may use malaria prophylaxis,
they're still at risk of malaria infection
Clinical picture of Cerebral
toxoplasmosis in AIDS ?
❊ most common infection of the CNS in
AIDS when the CD4 < 200
❊ focal neurological disturbance,
headache, confusion, fever and
convulsions
Dx of Cerebral toxoplasmosis in
AIDS ?
Toxoplasma cysts in the brain➜
multiple ring-enhancing masses with
surrounding oedema
Rx of Cerebral toxoplasmosis in
AIDS ?
Pyrimethamine — an antimalarial
medication
Sulfadiazine — an antibiotic used in
combination with pyrimethamine to treat
toxoplasmosis
+ Folic acid
What Is the Difference between HIV
and AIDS?
When a person is infected with the human immunodeficiency virus
(HIV) we say that he or she is "HIV positive" .A person who has
HIV is classified as having AIDS if one of two things happens:
❊ if the CD4 count has dropped below 200/cc, or
❊ if an HIV-related infection or HIV-related cancer develops.
A patient may have 200 CD4 cells or less and feel very healthy, but
he or she still has AIDS by definition.
MRCP Infectious Disease Revision notes
Dr.Sherif Elbadrawy
F Pt. returns from South Africa with confusion and
headache, no neck stiffness. She has a purpuric rash.
Malaria prophylaxis included mefloquine.most important
Ix ?
Lumbar puncture
Computed tomography (CT) scan
Multiple blood cultures
Malaria films
Clotting
Malaria films
Although the patient in the present
case has used malaria prophylaxis,
she is still at risk of malaria infection
Bacterial endocarditis is treated with penicillin. What
single additional step is undertaken to prevent penicillin
resistance?
Double the penicillin dose
Give a suitable B-lactam combination
Add macrolides
Add aminoglycosides
Interrupt penicillin treatment
Add aminoglycosides
Homosexual male with a lesion on his penis that was initially
nodular and painless➜ a heaped-up ulcer. Sampling ➜Biopsy:
large infected mononuclear cells
containing many Donovan bodies. What diagnosis fits best with
this clinical picture?
Penile carcinoma
Lymphogranuloma venereum
Chancroid
Genital herpes
Granuloma inguinaie
Granuloma inguinaie
Caused by➜ Klebsiella granulomatis
key features➜ painless indurated nodule➜heaped-
up ulcer + infected mononuclear cells containing
many Donovan bodies
Rx➜ tetracycline or ampicillin
25 y woman gives birth at full term to an otherwise well baby with
UNILATERAL MICROPHTHALMIA. She recalls a rash during the
first trimester of her pregnancy, but cannot remember any other
details of the rash. Which one of the following options is the most
likely causative agent?
☆ Syphilis
☆ Varicella zoster virus (VZV)
☆ Rubella
☆ Cytomegalovirus (CMV)
☆ Parvovirus B19
✱ Varicella zoster virus (VZV)
☆ The fetus develops episodes of shingles
that affect development of the involved
dermatome
☆ Congenital syphilis, CMV and rubella tend
to cause symmetrical problems
Etiology of mumps ? Paramyxovirus
MRCP Infectious Disease Revision notes
Dr.Sherif Elbadrawy
Etiology of Q fever ?
Coxiella burnetii
obligate intracellular bacteria & doesn't grow
on standard culture media.
man infection by inhalation or drinking
unpasteurized milk.
Criteria of Mycobacterium avium
complex (MAC) infection ?
✬ opportunist mycobacterium found in soil, water, dust, milk
✬ usually attacks patients with pre-existing lung disease ➜ COPD ,
bronchiectasis or immunosuppressed
✬ Dx➜ X-ray ➜cavitation,sputum C&S
✬ Rx➜ rifampicin and ethambutol for 24 months ± lsoniazid .
✬ if Rx failure ➜ add Ciprofloxacin or clarithromycin
✬ surgical resection of the disease area is an option in fit pts.
Etiology of Paragonimiasis ?
Paragonimus westermani is in the genus of
flatworms,( an infectious lung fluke).
lower lobe cavitating lesions, an eosinophilia
A granulomatous response to the eggs may
occur
DD from TB ➜lack of systemic symptoms
(fever, weight loss),-ve acid-fast bacilli
most common cause of failure of
antiretroviral Rx ?
Poor compliance to Rx.
Human T-cell lymphotrophic virus
type I is associated with which type of
cancer ?
acute T-cell lymphoma/leukaemia
MRCP Infectious Disease Revision notes
Dr.Sherif Elbadrawy
Human papilloma virus is associated
with which type of cancer ?
carcinoma of anal canal in
homosexual men
A 7-y boy who has come from lndia ➜ mild fever (37.5 °C),
maculopapular rash and pharyngitis. difficulty swallowing, even
his own saliva+ greyish membrane surrounding the tonsils+
regional lymphadenopathy. most likely diagnosis?
◆ Infectious mononucleosis
◆ Diphtheria
◆ Rubella
◆ Measles
◆ Streptococcus pyogenes
◆ Diphtheria
Which one of the following antigens is involved in
the entry of Plasmodium vivax into red cells?
Anti-D
Anti-S
Duffy
Kell
Kidd
Duffy
Constipation, mucopurulent anal
discharge, perianal pruritis and anal
bleeding and pain while opening
bowels in homosexual man ?
Think of Gonorrhoea (Smear reveals
intracellular diplococci).
Risk of vertical transmission in Fully
controlled HIV ?
perinatal transmission<1% and most
likelihood of transmission is
during delivery (risk minimized by
intrapartum zidovudine and 6 weeks
after birth to baby).
MRCP Infectious Disease Revision notes
Dr.Sherif Elbadrawy
Clinical picture of Tapeworm
infection ?
✼ Tape worms are made up of repeated segments
called proglottids (actively motile, elongating and
contracting)
✼ often present in faeces and are useful diagnostically
✼ Taenia solium (pork tapeworm) and Taenia
saginata (beef tapeworm) infection『Cysticercosis』
✼ niclosamide or praziquantel.
The larvae of which one of the following helminth species
undertakes a symptomatic travel through
the LUNG before the adult worms reside in the intestine?
Enterobfus vermfcularfs
Trfchfnella spfralfs
Ascaris lumbricoides
Trfchurfs trfchurfa
Taenfa sagfnat
Ascaris lumbricoides
Rx of choice for cryptococcus
neoformans ?
Amphotrecin B
Definition of Multidrug-resistant
tuberculosis ?
✪ resistance to rifampicin and isoniazid,
with or without resistance to other anti-TB
drugs.
✪ Rx at least 3 drugs to which the organism
is sensitive in-vitro for at least 9 months and
perhaps up to or beyond 24 months
Rx of Multidrug-resistant
tuberculosis ?
✪ has to be planned on an individual basis
✪ should only be carried out by physicians with
substantial experience in managing complex resistant
cases
✪ at least 3 drugs to which the organism is sensitive
in-vitro for at least 9 months and perhaps up to or
beyond 24 months
MRCP Infectious Disease Revision notes
Dr.Sherif Elbadrawy
percentages of untreated pts that will develop late
stages CNS & CVS complications ?
80%
90%
10%
30%
0%
30%
Difference Dengue and Malaria Fever
?
✷ Both spread by MOSQUITOES. Malaria is dt a parasite plasmodium whereas dengue is dt a
flavivirus (Dengue virus)
✷ Dengue is the fever é sudden onset and remains for long duration. It is reaches to the
temperature level of ≅ 40 ℃
✷ SEVERE HEADACHE mostly in the back portion of the eyes head is CHARACTERISTIC FOR
DENGUE.
✷ The dengue fever may disappear after the few days but sudden rise again with the problem of
SKIN RASHES (rash of Dengue isn't dependant on thrombocytopenia)(similar to measles)
✷ in malaria, fever is for shorter period. The symptoms of malaria are anemia, pain in the joints,
vomiting, sweating etc. There are 3 stages for the malaria symptoms. They are Cold stage,
Sweating stage and the last is again cold stage.
✷ Incubation period 10-15 days for malaria & 4-5 days for dengue.
in Dengue Lymphocytosis is seen although the total WBCs may be normal.
Selective serotonin reuptake inhibitor
(SSRI) withdrawal ?
SSRI withdrawal appears to be a particular
problem with paroxetine, which can lead to
symptoms of psychomotor agitation, poor
sleep and anxiety, vertigo and light
headedness. ± GIT symptoms.
avoided by slow tapering down the drug.
Dx of Schistosoma Hematobium ? a mid-day urine sample ➜ look for
the parasite
Clinical picture of Acute
schistosomiasis (Katayama's fever) ?
may occur weeks after the initial infection, especially by S.
mansoni and S. japonicum. Manifestations include:
Abdominal pain
Cough
Diarrhea
↟ Eosinophilia
Fever
Fatigue
Hepatosplenomegaly
MRCP Infectious Disease Revision notes
Dr.Sherif Elbadrawy
Elderly + orbital cellulitis +maxillary
sinusitis + C&S ➜ irregularly
branching septate hyphae ,Dx ?
Aspergillus
Clinical picture of Mucor mycosis ?
DM is the ppt factor.
Rhinocerebral Mucor mycosis is the most
common form.
necrotic black nasal turbinate, nasal
stufiness,facial pain & edema are
characteristic.
35 sailor ,painless swelling in the
sole of his foot , increase in
size,ulceration + yellow, white grains
on the surface Gm stain ➜Gm +ve
branching organism
Nocardia brasiliens
Mucocutaneous ulceration following
travel?
Leishmania brasiliensis
Start anti-retrovirals in HIV when
CD4
< 350
MRCP Infectious Disease Revision notes
Dr.Sherif Elbadrawy
Management of contacts of
meningococcal meningitis
ciprofloxacin is the drug of choice as
it is widely available and only
requires one dose, rifampicin may be
used
Management of contacts of
pneumococcal meninigitis
no prophylaxis is generally needed
Management of Meningococcal
Meningitis
pre-hospital (for example a GP surgery) and
meningococcal disease is suspected then
intramuscular benzylpenicillin , as long as
this doesn't delay transit to hospital ➜ If
immediate hypersensitivity reaction to
penicillin or to cephalosporins ➜ Give
Chloramphenicol.
Rx of Cellulitis
✯ Penicillin is the antibiotic of choice for group A streptococcal
infections.
✯ flucloxacillin is first-line treatment for mild/moderate cellulitis. [ stop
flucloxacillin if streptococcal infection is confirmed in patients with
cellulitis, dt the high sensitivity & change it with Penicillin]
✯ Clarithromycin or clindamycin is recommend in patients allergic to
penicillin.
Brucellosis
✯ > in Middle East and in farmers
✯ fever, malaise
✯ leukopenia
✯ hepatosplenomegaly
✯ sacroilitis: spinal tenderness
✯ osteomyelitis, infective endocarditis, orchitis,
meningoencephalitis
✯ screening ➜ Rose Bengal plate test
✯ Brucella serology is the best test for diagnosis
✯ Rx ➜ doxycycline and streptomycin
MRCP Infectious Disease Revision notes
Dr.Sherif Elbadrawy
Organisms which may colonise CF
patients
✯ Staphylococcus aureus
✯ Pseudomonas aeruginosa
✯ Burkholderia cepacia ➜
Pseudomonas cepacia
✯ Aspergillus
Bacteriostatic antibiotics
✯ chloramphenicol
✯ macrolides
✯ tetracyclines
✯ sulphonamides
✯ trimethoprim
Epididymo-orchitis
✯ infection of the epididymis ± testes ➜ pain and
swelling
✯ dt local spread of infections from the genital tract
(Chlamydia trachomatis and Neisseria gonorrhoeae) or
the bladder.
✯ U/L testicular pain and swelling ±urethral discharge
✯ DD is testicular torsion ➜ Pt < 20 years, severe pain
and acute onset
✯ Rx ➜ IM ceftriaxone stat + oral doxycyline for 2 weeks
HIV drugs, rule of thumb: NRTIs end in 'ine'
Pis: end in 'vir'
NNRTIs: nevirapine, efavirenz
MRCP Infectious Disease Revision notes
Dr.Sherif Elbadrawy

Weitere ähnliche Inhalte

Was ist angesagt?

HIV and TB coinfection
HIV and TB coinfectionHIV and TB coinfection
HIV and TB coinfectionswati2084
 
Internal Medicine Board Review - Dermatology Flashcards - by Knowmedge
Internal Medicine Board Review - Dermatology Flashcards - by KnowmedgeInternal Medicine Board Review - Dermatology Flashcards - by Knowmedge
Internal Medicine Board Review - Dermatology Flashcards - by KnowmedgeKnowmedge
 
Internal Medicine Board Review - Endocrinology Flashcards - by Knowmedge
Internal Medicine Board Review - Endocrinology Flashcards - by KnowmedgeInternal Medicine Board Review - Endocrinology Flashcards - by Knowmedge
Internal Medicine Board Review - Endocrinology Flashcards - by KnowmedgeKnowmedge
 
Arrhythmogenic right ventricular dysplasia
Arrhythmogenic right ventricular dysplasiaArrhythmogenic right ventricular dysplasia
Arrhythmogenic right ventricular dysplasiaDomina Petric
 
Internal Medicine Board Review - Rheumatology Flashcards - by Knowmedge
Internal Medicine Board Review - Rheumatology Flashcards -  by KnowmedgeInternal Medicine Board Review - Rheumatology Flashcards -  by Knowmedge
Internal Medicine Board Review - Rheumatology Flashcards - by KnowmedgeKnowmedge
 
500 single best answers in medicine
500 single best answers in medicine500 single best answers in medicine
500 single best answers in medicinehamadadodo
 
Internal Medicine Board Review - Cardiology Flashcards - by Knowmedge
Internal Medicine Board Review - Cardiology Flashcards - by KnowmedgeInternal Medicine Board Review - Cardiology Flashcards - by Knowmedge
Internal Medicine Board Review - Cardiology Flashcards - by KnowmedgeKnowmedge
 
Pulmonary Hypertension
Pulmonary HypertensionPulmonary Hypertension
Pulmonary HypertensionPrasad CSBR
 
Golden book for Medicine OSCE: First View
Golden book for Medicine OSCE: First ViewGolden book for Medicine OSCE: First View
Golden book for Medicine OSCE: First ViewMan B Paudyal
 
Sepsis 2017
Sepsis 2017Sepsis 2017
Sepsis 2017Badheeb
 
Antiretroviral therapy failure
Antiretroviral therapy failureAntiretroviral therapy failure
Antiretroviral therapy failureParvez Pathan
 
Module 3 opportunistic infections and hiv related conditi
Module 3  opportunistic infections and hiv  related  conditiModule 3  opportunistic infections and hiv  related  conditi
Module 3 opportunistic infections and hiv related conditiDavid Ngogoyo
 
Community acquired pneumonia
Community acquired pneumoniaCommunity acquired pneumonia
Community acquired pneumoniaami purohit
 

Was ist angesagt? (20)

HIV and TB coinfection
HIV and TB coinfectionHIV and TB coinfection
HIV and TB coinfection
 
Internal Medicine Board Review - Dermatology Flashcards - by Knowmedge
Internal Medicine Board Review - Dermatology Flashcards - by KnowmedgeInternal Medicine Board Review - Dermatology Flashcards - by Knowmedge
Internal Medicine Board Review - Dermatology Flashcards - by Knowmedge
 
Internal Medicine Board Review - Endocrinology Flashcards - by Knowmedge
Internal Medicine Board Review - Endocrinology Flashcards - by KnowmedgeInternal Medicine Board Review - Endocrinology Flashcards - by Knowmedge
Internal Medicine Board Review - Endocrinology Flashcards - by Knowmedge
 
Tb hiv-coinfection
Tb hiv-coinfectionTb hiv-coinfection
Tb hiv-coinfection
 
ANCA vasculitis
ANCA vasculitisANCA vasculitis
ANCA vasculitis
 
CHOOSING THE RIGHT ANTIBIOTIC ? Need for Antibiotic Policy
CHOOSING THE RIGHT ANTIBIOTIC ?Need for Antibiotic Policy         CHOOSING THE RIGHT ANTIBIOTIC ?Need for Antibiotic Policy
CHOOSING THE RIGHT ANTIBIOTIC ? Need for Antibiotic Policy
 
Arrhythmogenic right ventricular dysplasia
Arrhythmogenic right ventricular dysplasiaArrhythmogenic right ventricular dysplasia
Arrhythmogenic right ventricular dysplasia
 
Pneumocystis Pneumonia
Pneumocystis Pneumonia Pneumocystis Pneumonia
Pneumocystis Pneumonia
 
Internal Medicine Board Review - Rheumatology Flashcards - by Knowmedge
Internal Medicine Board Review - Rheumatology Flashcards -  by KnowmedgeInternal Medicine Board Review - Rheumatology Flashcards -  by Knowmedge
Internal Medicine Board Review - Rheumatology Flashcards - by Knowmedge
 
500 single best answers in medicine
500 single best answers in medicine500 single best answers in medicine
500 single best answers in medicine
 
A Case Of Dengue Fever with Myocarditis
A Case Of Dengue Fever with MyocarditisA Case Of Dengue Fever with Myocarditis
A Case Of Dengue Fever with Myocarditis
 
Internal Medicine Board Review - Cardiology Flashcards - by Knowmedge
Internal Medicine Board Review - Cardiology Flashcards - by KnowmedgeInternal Medicine Board Review - Cardiology Flashcards - by Knowmedge
Internal Medicine Board Review - Cardiology Flashcards - by Knowmedge
 
Endocarditis
EndocarditisEndocarditis
Endocarditis
 
Pulmonary Hypertension
Pulmonary HypertensionPulmonary Hypertension
Pulmonary Hypertension
 
Golden book for Medicine OSCE: First View
Golden book for Medicine OSCE: First ViewGolden book for Medicine OSCE: First View
Golden book for Medicine OSCE: First View
 
Sepsis 2017
Sepsis 2017Sepsis 2017
Sepsis 2017
 
Antiretroviral therapy failure
Antiretroviral therapy failureAntiretroviral therapy failure
Antiretroviral therapy failure
 
Management of TB 2019
Management of TB 2019Management of TB 2019
Management of TB 2019
 
Module 3 opportunistic infections and hiv related conditi
Module 3  opportunistic infections and hiv  related  conditiModule 3  opportunistic infections and hiv  related  conditi
Module 3 opportunistic infections and hiv related conditi
 
Community acquired pneumonia
Community acquired pneumoniaCommunity acquired pneumonia
Community acquired pneumonia
 

Andere mochten auch

MRCP Classical Diagnostics and Keywords.
MRCP Classical Diagnostics and Keywords.MRCP Classical Diagnostics and Keywords.
MRCP Classical Diagnostics and Keywords.Sherif Elbadrawy
 
Last minute mrcp1 revision
Last minute mrcp1 revisionLast minute mrcp1 revision
Last minute mrcp1 revisionSherif Elbadrawy
 
♕ Badrawy notes for mrcp ➜【basic science】
♕ Badrawy notes for mrcp ➜【basic science】♕ Badrawy notes for mrcp ➜【basic science】
♕ Badrawy notes for mrcp ➜【basic science】Sherif Elbadrawy
 
MRCP Classical Presentations
MRCP Classical PresentationsMRCP Classical Presentations
MRCP Classical PresentationsSherif Elbadrawy
 
Critical care revision notes
Critical care revision notesCritical care revision notes
Critical care revision notesSherif Elbadrawy
 
Pass medicine MRCP 2013
Pass medicine  MRCP 2013Pass medicine  MRCP 2013
Pass medicine MRCP 2013DrZahid Khan
 
Mrcp Radiology
Mrcp RadiologyMrcp Radiology
Mrcp Radiologykunalj000
 
Landmark Critical Care Clinical Trials
Landmark Critical Care Clinical TrialsLandmark Critical Care Clinical Trials
Landmark Critical Care Clinical TrialsSherif Elbadrawy
 
Intraaortic Baloon Counterpulsation
Intraaortic Baloon CounterpulsationIntraaortic Baloon Counterpulsation
Intraaortic Baloon CounterpulsationSherif Elbadrawy
 
Acid base balance interpretation
Acid base balance interpretationAcid base balance interpretation
Acid base balance interpretationSherif Elbadrawy
 
Dermatology osce slides
Dermatology osce slidesDermatology osce slides
Dermatology osce slidesHabrol Afzam
 
Tips on Central Venous Catheter & Pulmonary Artery Catheter.
Tips on Central Venous Catheter & Pulmonary Artery Catheter.Tips on Central Venous Catheter & Pulmonary Artery Catheter.
Tips on Central Venous Catheter & Pulmonary Artery Catheter.Sherif Elbadrawy
 

Andere mochten auch (20)

MRCP Classical Diagnostics and Keywords.
MRCP Classical Diagnostics and Keywords.MRCP Classical Diagnostics and Keywords.
MRCP Classical Diagnostics and Keywords.
 
Last minute mrcp1 revision
Last minute mrcp1 revisionLast minute mrcp1 revision
Last minute mrcp1 revision
 
♕ Badrawy notes for mrcp ➜【basic science】
♕ Badrawy notes for mrcp ➜【basic science】♕ Badrawy notes for mrcp ➜【basic science】
♕ Badrawy notes for mrcp ➜【basic science】
 
Dermatology for MRCP
Dermatology for MRCPDermatology for MRCP
Dermatology for MRCP
 
Classical Rx mrcp
Classical Rx mrcpClassical Rx mrcp
Classical Rx mrcp
 
MRCP Classical Presentations
MRCP Classical PresentationsMRCP Classical Presentations
MRCP Classical Presentations
 
Critical care revision notes
Critical care revision notesCritical care revision notes
Critical care revision notes
 
January 2014 MRCP1
January 2014 MRCP1January 2014 MRCP1
January 2014 MRCP1
 
Pass medicine MRCP 2013
Pass medicine  MRCP 2013Pass medicine  MRCP 2013
Pass medicine MRCP 2013
 
Mrcp 2 dermatology
Mrcp 2 dermatologyMrcp 2 dermatology
Mrcp 2 dermatology
 
Mrcp Radiology
Mrcp RadiologyMrcp Radiology
Mrcp Radiology
 
Landmark Critical Care Clinical Trials
Landmark Critical Care Clinical TrialsLandmark Critical Care Clinical Trials
Landmark Critical Care Clinical Trials
 
Intraaortic Baloon Counterpulsation
Intraaortic Baloon CounterpulsationIntraaortic Baloon Counterpulsation
Intraaortic Baloon Counterpulsation
 
Acid base balance interpretation
Acid base balance interpretationAcid base balance interpretation
Acid base balance interpretation
 
Blood Transfusion in ICU
Blood Transfusion in ICUBlood Transfusion in ICU
Blood Transfusion in ICU
 
A-a Gradient simplified
A-a Gradient simplifiedA-a Gradient simplified
A-a Gradient simplified
 
Dermatology osce slides
Dermatology osce slidesDermatology osce slides
Dermatology osce slides
 
Mrcp Part 2 Witten Exam
Mrcp Part 2 Witten ExamMrcp Part 2 Witten Exam
Mrcp Part 2 Witten Exam
 
Tips on Central Venous Catheter & Pulmonary Artery Catheter.
Tips on Central Venous Catheter & Pulmonary Artery Catheter.Tips on Central Venous Catheter & Pulmonary Artery Catheter.
Tips on Central Venous Catheter & Pulmonary Artery Catheter.
 
Dermatology Foreign Certificates
Dermatology Foreign CertificatesDermatology Foreign Certificates
Dermatology Foreign Certificates
 

Ähnlich wie MRCP Infectious disease notes.

Ähnlich wie MRCP Infectious disease notes. (20)

Needle stick Injury
Needle stick Injury Needle stick Injury
Needle stick Injury
 
Opportunistic Infections in HIV
Opportunistic Infections in HIVOpportunistic Infections in HIV
Opportunistic Infections in HIV
 
Meningitis with HIV AIDS
Meningitis with HIV AIDSMeningitis with HIV AIDS
Meningitis with HIV AIDS
 
Human cmv in immuncompromised and congential patients 2019
Human cmv in immuncompromised and congential patients  2019Human cmv in immuncompromised and congential patients  2019
Human cmv in immuncompromised and congential patients 2019
 
Sepsis without focus
Sepsis without focusSepsis without focus
Sepsis without focus
 
AIDS
AIDSAIDS
AIDS
 
HIV by dr mohammed hussien
HIV by  dr mohammed hussienHIV by  dr mohammed hussien
HIV by dr mohammed hussien
 
Acute liver disease
Acute liver diseaseAcute liver disease
Acute liver disease
 
Hepatitis
HepatitisHepatitis
Hepatitis
 
HEPATITIS.ppt
HEPATITIS.pptHEPATITIS.ppt
HEPATITIS.ppt
 
Know about jaundice and its Treatment, Dr. Sharda Jain , Dr. jyoti Agarwal, D...
Know about jaundice and its Treatment, Dr. Sharda Jain , Dr. jyoti Agarwal, D...Know about jaundice and its Treatment, Dr. Sharda Jain , Dr. jyoti Agarwal, D...
Know about jaundice and its Treatment, Dr. Sharda Jain , Dr. jyoti Agarwal, D...
 
Occupational
OccupationalOccupational
Occupational
 
Hiv Aids
Hiv  AidsHiv  Aids
Hiv Aids
 
Hiv in pregnancy
Hiv in pregnancyHiv in pregnancy
Hiv in pregnancy
 
The human immunodeficiency virus (HIV)
The human immunodeficiency virus (HIV)The human immunodeficiency virus (HIV)
The human immunodeficiency virus (HIV)
 
Hiv 140123032347-phpapp02
Hiv 140123032347-phpapp02Hiv 140123032347-phpapp02
Hiv 140123032347-phpapp02
 
hiv infection.pptx
hiv infection.pptxhiv infection.pptx
hiv infection.pptx
 
Viral hepatitis by dr abas Reshi.ppt.pdf
Viral hepatitis by dr abas Reshi.ppt.pdfViral hepatitis by dr abas Reshi.ppt.pdf
Viral hepatitis by dr abas Reshi.ppt.pdf
 
Dunc+hep+hiv+for+hull+2013
Dunc+hep+hiv+for+hull+2013Dunc+hep+hiv+for+hull+2013
Dunc+hep+hiv+for+hull+2013
 
HIV and SURGERY(adesiyakan)
HIV and SURGERY(adesiyakan)HIV and SURGERY(adesiyakan)
HIV and SURGERY(adesiyakan)
 

Mehr von Sherif Elbadrawy

Ann thoracmed 2015 Near fatal asthma
Ann thoracmed 2015 Near fatal asthmaAnn thoracmed 2015 Near fatal asthma
Ann thoracmed 2015 Near fatal asthmaSherif Elbadrawy
 
【مناسك الحج خطوة خطوة】
【مناسك الحج خطوة خطوة】【مناسك الحج خطوة خطوة】
【مناسك الحج خطوة خطوة】Sherif Elbadrawy
 
ARDS 【A simplified evidence based approach】
ARDS 【A simplified evidence based approach】ARDS 【A simplified evidence based approach】
ARDS 【A simplified evidence based approach】Sherif Elbadrawy
 

Mehr von Sherif Elbadrawy (6)

Basic Life Support
Basic Life SupportBasic Life Support
Basic Life Support
 
Ann thoracmed 2015 Near fatal asthma
Ann thoracmed 2015 Near fatal asthmaAnn thoracmed 2015 Near fatal asthma
Ann thoracmed 2015 Near fatal asthma
 
ICU Trials summary
ICU Trials summaryICU Trials summary
ICU Trials summary
 
【مناسك الحج خطوة خطوة】
【مناسك الحج خطوة خطوة】【مناسك الحج خطوة خطوة】
【مناسك الحج خطوة خطوة】
 
Brain death in ICU
Brain death in ICUBrain death in ICU
Brain death in ICU
 
ARDS 【A simplified evidence based approach】
ARDS 【A simplified evidence based approach】ARDS 【A simplified evidence based approach】
ARDS 【A simplified evidence based approach】
 

Kürzlich hochgeladen

PULMONARY EDEMA AND ITS MANAGEMENT.pdf
PULMONARY EDEMA AND  ITS  MANAGEMENT.pdfPULMONARY EDEMA AND  ITS  MANAGEMENT.pdf
PULMONARY EDEMA AND ITS MANAGEMENT.pdfDolisha Warbi
 
LUNG TUMORS AND ITS CLASSIFICATIONS.pdf
LUNG TUMORS AND ITS  CLASSIFICATIONS.pdfLUNG TUMORS AND ITS  CLASSIFICATIONS.pdf
LUNG TUMORS AND ITS CLASSIFICATIONS.pdfDolisha Warbi
 
Music Therapy's Impact in Palliative Care| IAPCON2024| Dr. Tara Rajendran
Music Therapy's Impact in Palliative Care| IAPCON2024| Dr. Tara RajendranMusic Therapy's Impact in Palliative Care| IAPCON2024| Dr. Tara Rajendran
Music Therapy's Impact in Palliative Care| IAPCON2024| Dr. Tara RajendranTara Rajendran
 
See the 2,456 pharmacies on the National E-Pharmacy Platform
See the 2,456 pharmacies on the National E-Pharmacy PlatformSee the 2,456 pharmacies on the National E-Pharmacy Platform
See the 2,456 pharmacies on the National E-Pharmacy PlatformKweku Zurek
 
Report Back from SGO: What’s New in Uterine Cancer?.pptx
Report Back from SGO: What’s New in Uterine Cancer?.pptxReport Back from SGO: What’s New in Uterine Cancer?.pptx
Report Back from SGO: What’s New in Uterine Cancer?.pptxbkling
 
Informed Consent Empowering Healthcare Decision-Making.pptx
Informed Consent Empowering Healthcare Decision-Making.pptxInformed Consent Empowering Healthcare Decision-Making.pptx
Informed Consent Empowering Healthcare Decision-Making.pptxSasikiranMarri
 
VarSeq 2.6.0: Advancing Pharmacogenomics and Genomic Analysis
VarSeq 2.6.0: Advancing Pharmacogenomics and Genomic AnalysisVarSeq 2.6.0: Advancing Pharmacogenomics and Genomic Analysis
VarSeq 2.6.0: Advancing Pharmacogenomics and Genomic AnalysisGolden Helix
 
call girls in munirka DELHI 🔝 >༒9540349809 🔝 genuine Escort Service 🔝✔️✔️
call girls in munirka  DELHI 🔝 >༒9540349809 🔝 genuine Escort Service 🔝✔️✔️call girls in munirka  DELHI 🔝 >༒9540349809 🔝 genuine Escort Service 🔝✔️✔️
call girls in munirka DELHI 🔝 >༒9540349809 🔝 genuine Escort Service 🔝✔️✔️saminamagar
 
Let's Talk About It: To Disclose or Not to Disclose?
Let's Talk About It: To Disclose or Not to Disclose?Let's Talk About It: To Disclose or Not to Disclose?
Let's Talk About It: To Disclose or Not to Disclose?bkling
 
COVID-19 (NOVEL CORONA VIRUS DISEASE PANDEMIC ).pptx
COVID-19  (NOVEL CORONA  VIRUS DISEASE PANDEMIC ).pptxCOVID-19  (NOVEL CORONA  VIRUS DISEASE PANDEMIC ).pptx
COVID-19 (NOVEL CORONA VIRUS DISEASE PANDEMIC ).pptxBibekananda shah
 
POST NATAL EXERCISES AND ITS IMPACT.pptx
POST NATAL EXERCISES AND ITS IMPACT.pptxPOST NATAL EXERCISES AND ITS IMPACT.pptx
POST NATAL EXERCISES AND ITS IMPACT.pptxvirengeeta
 
METHODS OF ACQUIRING KNOWLEDGE IN NURSING.pptx by navdeep kaur
METHODS OF ACQUIRING KNOWLEDGE IN NURSING.pptx by navdeep kaurMETHODS OF ACQUIRING KNOWLEDGE IN NURSING.pptx by navdeep kaur
METHODS OF ACQUIRING KNOWLEDGE IN NURSING.pptx by navdeep kaurNavdeep Kaur
 
PERFECT BUT PAINFUL TKR -ROLE OF SYNOVECTOMY.pptx
PERFECT BUT PAINFUL TKR -ROLE OF SYNOVECTOMY.pptxPERFECT BUT PAINFUL TKR -ROLE OF SYNOVECTOMY.pptx
PERFECT BUT PAINFUL TKR -ROLE OF SYNOVECTOMY.pptxdrashraf369
 
Lippincott Microcards_ Microbiology Flash Cards-LWW (2015).pdf
Lippincott Microcards_ Microbiology Flash Cards-LWW (2015).pdfLippincott Microcards_ Microbiology Flash Cards-LWW (2015).pdf
Lippincott Microcards_ Microbiology Flash Cards-LWW (2015).pdfSreeja Cherukuru
 
SYNDESMOTIC INJURY- ANATOMICAL REPAIR.pptx
SYNDESMOTIC INJURY- ANATOMICAL REPAIR.pptxSYNDESMOTIC INJURY- ANATOMICAL REPAIR.pptx
SYNDESMOTIC INJURY- ANATOMICAL REPAIR.pptxdrashraf369
 
call girls in aerocity DELHI 🔝 >༒9540349809 🔝 genuine Escort Service 🔝✔️✔️
call girls in aerocity DELHI 🔝 >༒9540349809 🔝 genuine Escort Service 🔝✔️✔️call girls in aerocity DELHI 🔝 >༒9540349809 🔝 genuine Escort Service 🔝✔️✔️
call girls in aerocity DELHI 🔝 >༒9540349809 🔝 genuine Escort Service 🔝✔️✔️saminamagar
 
History and Development of Pharmacovigilence.pdf
History and Development of Pharmacovigilence.pdfHistory and Development of Pharmacovigilence.pdf
History and Development of Pharmacovigilence.pdfSasikiranMarri
 
SWD (Short wave diathermy)- Physiotherapy.ppt
SWD (Short wave diathermy)- Physiotherapy.pptSWD (Short wave diathermy)- Physiotherapy.ppt
SWD (Short wave diathermy)- Physiotherapy.pptMumux Mirani
 
Statistical modeling in pharmaceutical research and development.
Statistical modeling in pharmaceutical research and development.Statistical modeling in pharmaceutical research and development.
Statistical modeling in pharmaceutical research and development.ANJALI
 
Presentació "Real-Life VR Integration for Mild Cognitive Impairment Rehabilit...
Presentació "Real-Life VR Integration for Mild Cognitive Impairment Rehabilit...Presentació "Real-Life VR Integration for Mild Cognitive Impairment Rehabilit...
Presentació "Real-Life VR Integration for Mild Cognitive Impairment Rehabilit...Badalona Serveis Assistencials
 

Kürzlich hochgeladen (20)

PULMONARY EDEMA AND ITS MANAGEMENT.pdf
PULMONARY EDEMA AND  ITS  MANAGEMENT.pdfPULMONARY EDEMA AND  ITS  MANAGEMENT.pdf
PULMONARY EDEMA AND ITS MANAGEMENT.pdf
 
LUNG TUMORS AND ITS CLASSIFICATIONS.pdf
LUNG TUMORS AND ITS  CLASSIFICATIONS.pdfLUNG TUMORS AND ITS  CLASSIFICATIONS.pdf
LUNG TUMORS AND ITS CLASSIFICATIONS.pdf
 
Music Therapy's Impact in Palliative Care| IAPCON2024| Dr. Tara Rajendran
Music Therapy's Impact in Palliative Care| IAPCON2024| Dr. Tara RajendranMusic Therapy's Impact in Palliative Care| IAPCON2024| Dr. Tara Rajendran
Music Therapy's Impact in Palliative Care| IAPCON2024| Dr. Tara Rajendran
 
See the 2,456 pharmacies on the National E-Pharmacy Platform
See the 2,456 pharmacies on the National E-Pharmacy PlatformSee the 2,456 pharmacies on the National E-Pharmacy Platform
See the 2,456 pharmacies on the National E-Pharmacy Platform
 
Report Back from SGO: What’s New in Uterine Cancer?.pptx
Report Back from SGO: What’s New in Uterine Cancer?.pptxReport Back from SGO: What’s New in Uterine Cancer?.pptx
Report Back from SGO: What’s New in Uterine Cancer?.pptx
 
Informed Consent Empowering Healthcare Decision-Making.pptx
Informed Consent Empowering Healthcare Decision-Making.pptxInformed Consent Empowering Healthcare Decision-Making.pptx
Informed Consent Empowering Healthcare Decision-Making.pptx
 
VarSeq 2.6.0: Advancing Pharmacogenomics and Genomic Analysis
VarSeq 2.6.0: Advancing Pharmacogenomics and Genomic AnalysisVarSeq 2.6.0: Advancing Pharmacogenomics and Genomic Analysis
VarSeq 2.6.0: Advancing Pharmacogenomics and Genomic Analysis
 
call girls in munirka DELHI 🔝 >༒9540349809 🔝 genuine Escort Service 🔝✔️✔️
call girls in munirka  DELHI 🔝 >༒9540349809 🔝 genuine Escort Service 🔝✔️✔️call girls in munirka  DELHI 🔝 >༒9540349809 🔝 genuine Escort Service 🔝✔️✔️
call girls in munirka DELHI 🔝 >༒9540349809 🔝 genuine Escort Service 🔝✔️✔️
 
Let's Talk About It: To Disclose or Not to Disclose?
Let's Talk About It: To Disclose or Not to Disclose?Let's Talk About It: To Disclose or Not to Disclose?
Let's Talk About It: To Disclose or Not to Disclose?
 
COVID-19 (NOVEL CORONA VIRUS DISEASE PANDEMIC ).pptx
COVID-19  (NOVEL CORONA  VIRUS DISEASE PANDEMIC ).pptxCOVID-19  (NOVEL CORONA  VIRUS DISEASE PANDEMIC ).pptx
COVID-19 (NOVEL CORONA VIRUS DISEASE PANDEMIC ).pptx
 
POST NATAL EXERCISES AND ITS IMPACT.pptx
POST NATAL EXERCISES AND ITS IMPACT.pptxPOST NATAL EXERCISES AND ITS IMPACT.pptx
POST NATAL EXERCISES AND ITS IMPACT.pptx
 
METHODS OF ACQUIRING KNOWLEDGE IN NURSING.pptx by navdeep kaur
METHODS OF ACQUIRING KNOWLEDGE IN NURSING.pptx by navdeep kaurMETHODS OF ACQUIRING KNOWLEDGE IN NURSING.pptx by navdeep kaur
METHODS OF ACQUIRING KNOWLEDGE IN NURSING.pptx by navdeep kaur
 
PERFECT BUT PAINFUL TKR -ROLE OF SYNOVECTOMY.pptx
PERFECT BUT PAINFUL TKR -ROLE OF SYNOVECTOMY.pptxPERFECT BUT PAINFUL TKR -ROLE OF SYNOVECTOMY.pptx
PERFECT BUT PAINFUL TKR -ROLE OF SYNOVECTOMY.pptx
 
Lippincott Microcards_ Microbiology Flash Cards-LWW (2015).pdf
Lippincott Microcards_ Microbiology Flash Cards-LWW (2015).pdfLippincott Microcards_ Microbiology Flash Cards-LWW (2015).pdf
Lippincott Microcards_ Microbiology Flash Cards-LWW (2015).pdf
 
SYNDESMOTIC INJURY- ANATOMICAL REPAIR.pptx
SYNDESMOTIC INJURY- ANATOMICAL REPAIR.pptxSYNDESMOTIC INJURY- ANATOMICAL REPAIR.pptx
SYNDESMOTIC INJURY- ANATOMICAL REPAIR.pptx
 
call girls in aerocity DELHI 🔝 >༒9540349809 🔝 genuine Escort Service 🔝✔️✔️
call girls in aerocity DELHI 🔝 >༒9540349809 🔝 genuine Escort Service 🔝✔️✔️call girls in aerocity DELHI 🔝 >༒9540349809 🔝 genuine Escort Service 🔝✔️✔️
call girls in aerocity DELHI 🔝 >༒9540349809 🔝 genuine Escort Service 🔝✔️✔️
 
History and Development of Pharmacovigilence.pdf
History and Development of Pharmacovigilence.pdfHistory and Development of Pharmacovigilence.pdf
History and Development of Pharmacovigilence.pdf
 
SWD (Short wave diathermy)- Physiotherapy.ppt
SWD (Short wave diathermy)- Physiotherapy.pptSWD (Short wave diathermy)- Physiotherapy.ppt
SWD (Short wave diathermy)- Physiotherapy.ppt
 
Statistical modeling in pharmaceutical research and development.
Statistical modeling in pharmaceutical research and development.Statistical modeling in pharmaceutical research and development.
Statistical modeling in pharmaceutical research and development.
 
Presentació "Real-Life VR Integration for Mild Cognitive Impairment Rehabilit...
Presentació "Real-Life VR Integration for Mild Cognitive Impairment Rehabilit...Presentació "Real-Life VR Integration for Mild Cognitive Impairment Rehabilit...
Presentació "Real-Life VR Integration for Mild Cognitive Impairment Rehabilit...
 

MRCP Infectious disease notes.

  • 1. MRCP Infectious Disease Notes BY DR. SHERIF ELBADRAWY MRCP Infectious Disease Revision notes Dr.Sherif Elbadrawy
  • 2. Causes of Bacterial meningitis ? - Streptococcus pneumoniae - Neisserie meningitidis - Listeria rnonocytogenes Causes of Otitis media ? - Streptococcus pneumonia Causes of Community-acquired- Pneumonia ? - Streptococcus pneumonae - Haemophitus influenzae - Staphylococcus aureus Causes of Atypical Pneumonia ? - Mycoptasma pneumoniae - Chlamydia pneumoniae - Legioneiia pneumophiia Causes of Eye infections ? - Staphylococcus aureus - Neissoria gonorrhoeae - Chlamydia trachomatis MRCP Infectious Disease Revision notes Dr.Sherif Elbadrawy Dr. Sherif Elbadrawy Digitally signed by Dr. Sherif Elbadrawy DN: cn=Dr. Sherif Elbadrawy, o=Westmead Hospital, ou, email=sherif_badrawy@yahoo.com, c=AU Date: 2017.01.18 19:30:37 +11'00'
  • 3. Causes of Sexually transmitted diseases ? - Chlamydia trachomatis - Neisseria gonorrhoeae - Treponema pallidum - Ureaplasma urealyticum - Haemophilus ducreyi Causes of Skin infections ? - Staphylococcus aureus - Streptococcus pyogenes - Pseudomonas aeruginosa Causes of Urinary tract infections ? - Escherichia col - Other Enterobactedaceae - Staphylococcus saprophyticus - Pseudomonas aeruginosa Gram (P)ositive cocci ? sta(P)hylococci + stre(P)tococci (including enterococci) Gram (N)egative cocci ? (N)eisseria meningitidis +(N)eisseria gonorrhoeae, also Moraxella MRCP Infectious Disease Revision notes Dr.Sherif Elbadrawy
  • 4. Gram positive bacilli ? ☀ Actinomyces ☀ Bacillus antracis (anthrax) ☀ Clostridium ☀ Diphtheria: Corynebacterium diphtheriae ☀ Listeria monocytogenes Antibiotics with anti-anaerobic activity ? ☆ Penicillins ☆ Cephalosporins (except ceftazidime) ☆ Metronidazole ☆ Clindamycin ☆ Erythromycin ☆ Tetracycline Antibiotics with NO anti-anaerobic activity ? ☆ Gentamicin ☆ Ciprofloxacin ☆ Ceftazidime Incubation Period Longer than 3 weeks ? ☆ Infectious mononucleosis ☆ Cytomegalovirus ☆ Viral hepatitis ☆ HIV Incubation Period 2 - 3 weeks ? ☆ Mumps ☆ Rubella ☆ Chickenpox MRCP Infectious Disease Revision notes Dr.Sherif Elbadrawy
  • 5. Incubation Period 1 - 2 weeks ? ☆ Malaria ☆ Measles ☆ Dengue fever ☆ Typhoid Incubation Period Less than 1 week ? ☆ Scarlet fever ☆ Influenza ☆ Diphtheria ☆ Meningococcus Vaccine using Fragment/Extracts of the organism or virus ?(recombinant DNA technology) ✰ diphtheria ✰ pertussis ('acellular' vaccine) ✰ heptitis B ✰ meningococcus, pneumococcus, hemophilus Composition of cholera Vaccine ? inactivated Inaba and Ogawa strains of Vibrio cholerae together with recombinant B-subunit of the cholera toxin Post-Exposure Prophylaxis in Hepatitis A ? IV IG or hepatitis A vaccine MRCP Infectious Disease Revision notes Dr.Sherif Elbadrawy
  • 6. Post-Exposure Prophylaxis in Hepatitis B ? ☠ HBsAg positive source :- ✾ a known responder to HBV vaccine→ a booster dose should be given. ✾ If in the process of being vaccinated or are a nonresponder→ IV IG + vaccine ☠ Unknown source :- ✾ a known responder to HBV vaccine→a booster dose of HBV vaccine. ✾ If a nonresponder→ IV IG + vaccine ✾ If in the process of being vaccinated →an accelerated course of HBV vaccine Post-Exposure Prophylaxis in Hepatitis C ? Monthly PCR - if seroconversion ➜ interferon +/- ribavirin Post-Exposure Prophylaxis in HIV ? ✿ A combination of oral antiretrovirals (e.g. Tenofovir, emtricitabine, lopinavir and ritonavir) ASAP (i.e. Within 1-2 hours, but may be started up to 72 hours following exposure) for 4 weeks ✿ Serological testing at 12 weeks following completion of post-exposure prophylaxis ✿ ↓ risk of transmission by 80% Post-Exposure Prophylaxis in Varicella zoster ? ✿ Antibody Testing ✿ VZIG for IgG negative pregnant women /immunosuppressed Indictions of Tetanus vaccine ? ✿ given in the UK as part of the routine immunisation ☀ high-risk wounds (e.g. compound fractures, delayed surgical intervention, severely devitalised tissue) :- ✿ Vaccinated➜ IM IG. ✿ Incomplete or unknown vaccination history ➜ IM IG + a dose of tetanus vaccine. MRCP Infectious Disease Revision notes Dr.Sherif Elbadrawy
  • 7. Indictions of Meningococcal Vaccine ? Meningococcal vaccine is routinely indicated at the age 11 visit. The vaccine is also indicated for adults with the following circumstances: • Asplenia • Terminal complement deficiency • Military recruits • Residents of college dormitories • Travelers to Mecca or Medina in Saudi Arabia for the Hajj (pilgrimage) Tetanus ➜ Causative organism,Mode of infection, mechanism of toxicity ? Clostridium tetani➜Tetanus spores in soil → introduced into the body from a wound➜tetanospasmin exotoxin➜prevents release of GABA Clinical picture of Tetanus ? ❒ Prodrome fever, lethargy, headache ❒ Trismus (lockjaw) ❒ Risus sardonicus ❒ Opisthotonus (arched back, hyperextended neck) ❒ Spasms (e.g. Dysphagia) Rx of Tetanus ? ❁ MV & ms relaxants ❁ IM IG. for high-risk wounds (e.g. Compound fractures, severely devitalised tissue) ❁ Metronidazole is DOCH When to suspect HIV seroconversion ? ☹ a glandular fever 3-12 weeks after infection ☹ symptomatic in 60-80% of patients (Man returns from trip abroad with maculopapular rash and flu- like illness - think HIV seroconversion). ☹ For questions involving businessmen always consider sexually transmitted infections. MRCP Infectious Disease Revision notes Dr.Sherif Elbadrawy
  • 8. Clinical picture of HIV seroconversion (glandular fever)? ☆ HIV seroconversion resembles infection with cytomegalovirus, toxoplasmosis and the Epstein-Barr virus ◥ Lymphadenopathy ◥ Diarrhoea ◥ Maculopapular rash ◥ Sore throat ◥ Malaise, myalgia, arthralgia ◥ Mouth ulcers ◥ Rarely meningoencephalitis Dx of HIV seroconversion ? ◥ ELISA, confirmed by Western blotting ◥ Antibodies to HIV may not be present ◥ HIV PCR and p24 antigen tests can confirm diagnosis ◥ The CD4+ count is not used for diagnostic purposes immunological changes in progressive HIV ? ❀ Reduction in CD4 count ❀ Increase B2-Microglobulin (IBM) ❀ Decrease IL-2 production (DIL=DELL) 〘 IBM & DELL 〙 ❀ Polyclonal B-cell activation ❀ ↓ NK cell function ❀ ↓ delayed hypersensitivity responses Contraindicated Vaccines in HIV +ve pt ? (Cholera SABIN's Beautiful, INcredible!) (all are live attenuated except for cholera) ❀ Cholera ❀ SABIN's = Sabin's polio virus (oral) ❀ B = BCG ❀ IN = influenza (intranasal) Vaccines that can be used if CD4 > 200 in HIV +ve pt ? (Varicella Yellow MMR) ❀ Varicella ❀ Yellow Fever ❀ MMR MRCP Infectious Disease Revision notes Dr.Sherif Elbadrawy
  • 9. Causes of Diarrhea in HIV +ve pt ? ❂ CRYPTOSPORIDIUM (most common)+ other protozoa. ❂ Effects of the virus itself (HIV enteritis) ❂ Cytomegalovirus ❂ Mycobacterium avium intracellulare ❂ Giardia Cryptosporidium infection in HIV +ve pt ? ☆ MCC of DIARRHEA in HIV patients. ☆ incubation period of 7 days. ☆ Ziehl-Neelsen stain of the stool ➜ characteristic red cysts of Cryptosporidium. ☆ Difficult to Rx (supportive therapy) Mycobacterium avium intracellulare infection in HIV +ve pt ? ❐ CD4 < 50. ❐ fever, sweats, abdominal pain & DIARRHEA ± hepatomegaly and ↑LFTs ❐ Dx ➜ blood cultures and BM aspiration ❐ Rx ➜ rifampicin, ethambutol and clarithromycin Factors which ↓ vertical transmission of HIV ? ✪ Maternal antiretroviral therapy ✪ Neonatal antiretroviral therapy ✪ Mode of delivery (caesarean section) ✪ Infant feeding (bottle feeding) Should pregnant women do screening for HIV ? 【YES】 NICE guidelines recommend offering HIV screening to all pregnant women MRCP Infectious Disease Revision notes Dr.Sherif Elbadrawy
  • 10. Whom should be offered antiretroviral therapy in pregnant HIV +ve pt ? ✪ 【ALL】pregnant HIV +ve pt ✪ If not using before ➜ start at 28 and 32 weeks of gestation ➜ continue intrapartum. Mode of delivery in pregnant HIV +ve pt ? ✪ Elective CS ✪ zidovudine infusion before CS ✪【Zidovudine 】 orally to the neonate for 4 to 6 weeks. ✪ AVOID breast feed Best Neonatal antiretroviral therapy to protect against HIV of the baby in HIV +ve mother ? ✪【Zidovudine 】 orally to the neonate for 4 to 6 weeks. ✪ AVOID breast feed Etiology of Kaposi's sarcoma ? HHV-8 (Human Herpes Virus 8) 【purple papules】 or plaques on the:- ✿ skin➜ ± ulcerate ✿ mucosa (e.g. GIT and respiratory tract)➜ massive hemoptysis and pleural effusion ✿ HAART➜ ↑CD4 count ➜ improves Kaposi's sarcoma ✿ Radiotherapy + Surgical resection may be used ✿ Intralesional injections of vincristine or interferon Clinical picture of Kaposi's sarcoma ? 【purple papules】 or plaques on the:- ✿ skin➜ ± ulcerate ✿ mucosa (e.g. GIT and respiratory tract)➜ massive hemoptysis and pleural effusion MRCP Infectious Disease Revision notes Dr.Sherif Elbadrawy
  • 11. Rx of Kaposi Sarcoma ? ✿ HAART➜ ↑CD4 count ➜ improves Kaposi's sarcoma ✿ Radiotherapy + Surgical resection may be used ✿ Intralesional injections of vincristine or interferon most common opportunistic infection in AIDS ? PCP Which HIV pt. to give PCP prophylaxis ? All patients with a CD4 count < 200/mm³ Dx of PCP ? ✿ most common opportunistic infection in AIDS ✿ All patients with a CD4 count < 200 ➜ PCP prophylaxis ✿ Clinical pulmonary ➜ non-specific(Dyspnea,Dry cough,Fever,Very few chest signs) Exercise-induced desaturation ✿ Extrapulmonary ➜ hepatosplenomegaly ,Lymphadenopathy ,Choroid lesions ✿ CXR ➜ typically【 B/L INTERSTITIAL PULMONARY INFILTRATES】 but can present with other x-ray findings e.g. lobar consolidation. May be normal ✿ Sputum ➜ fails to Dx PCP ✿ BAL➜Dx PCP (SILVER STAIN) Rx of PCP ? ✿ Co-trimoxazole ✿ IV pentamidine in severe cases ✿ Steroids if hypoxic (if pO2 < 9.3kpa) MRCP Infectious Disease Revision notes Dr.Sherif Elbadrawy
  • 12. The most common cause of biliary disease in patients with HIV is ? 【sclerosing cholangitis】 due to infections such as CMV, Cryptosporidium and Microsporidia Pancreatitis é HIV infection dt ? anti-retroviral treatment 【didanosine】or by opportunistic infections e.g. CMV. Definition of Immune reconstitution inflammatory syndrome (IRIS) ? ( ‫ﻋ‬‫ﺎ‬‫د‬ ‫ﻟ‬‫ﯾ‬‫ﻧ‬‫ﺗ‬‫ﻘ‬‫م‬ ) when the immune system begins to recover (few weeks after starting Anti-retroviral Rx)➜ responds to a previously acquired opportunistic infection with an overwhelming inflammatory response ➜paradoxically makes the symptoms of infection worse. ✼【TB】 and cryptococcal meningitis. Infections most commonly associated with IRIS ? 【TB】 and cryptococcal meningitis. Bacterial Meningitis CSF ? ✯ Cloudy ✯ Low Glucose (< 1/2 plasma) ✯ High Protein (> 1 g/l) ✯ 10 - 5,000 POLYMORPHS MRCP Infectious Disease Revision notes Dr.Sherif Elbadrawy
  • 13. Viral Meningitis CSF ? ✯ Clear/cloudy ✯ Normal Glucose ➜ mumps & herpes encephalitis ± low glucose ✯ Normal/raised Protein ✯ 15 - 1,000 LYMPHOCYTES TB Meningitis CSF ? ✯ Fibrin web ✯ Low Glucose (< 1/2 plasma) ✯ High Protein (> 1 g/l) ✯ 15 - 1,000 LYMPHOCYTES ✯ PCR > better than Ziehl-Neelsen stain Contraindications to LP ? ● Possible ↑ ICP ●Thrombocytopenia or other bleeding diathesis (including ongoing anticoagulant therapy) ●Suspected spinal epidural abscess The most important α hemolytic Streptococcus ? ● Streptococcus pneumoniae (pneumococcus). ● a common cause of ➜ pneumonia, meningitis and otitis media. Another example is Strept viridans. ● Penicillin is the antibiotic of choice The most important Group A β hemolytic Streptococcus ? ● Streptococcus pyogenes ● erysipelas, impetigo, cellulitis, type 2 necrotizing fasciitis & pharyngitis/tonsillitis ● Immunological reactions➜ rheumatic fever or post-streptococcal glomerulonephritis ● Erythrogenic toxins ➜ scarlet fever ● Penicillin is the antibiotic of choice MRCP Infectious Disease Revision notes Dr.Sherif Elbadrawy
  • 14. The most important Group B β hemolytic Streptococcus ? Streptococcus agalactiae ➜ neonatal meningitis and septicemia CDC diagnostic criteria for Staphylococcal toxic shock syndrome ? ✰ Fever: temperature > 38.9ºc ✰ Hypotension: SBP < 90 mmHg ✰ RASH Diffuse erythematous ✰ ±infected tampons ✰ Desquamation of rash, esp of the palms & soles ✰ ≥ 3 organ systems Types of Necrotising fasciitis according to causative organism ? ✰ Type 1 is caused by mixed aerobes & anaerobes (often occurs post- surgery in diabetics) ✰ Type 2 is caused by Streptococcus pyogenes Clinical picture of Necrotising fasciitis ? ✿ progressive destruction of fascia and SC fat; ms is frequently spared dt ↑ blood supply. Acute ➜ skin color from red-purple to patches of blue-gray ➜Extremely tender ➜ skin breakdown with bullae ➜ frank gangrene Rx of Necrotising fasciitis ? ✿ Urgent surgical debridement ✿ IV antibiotics MRCP Infectious Disease Revision notes Dr.Sherif Elbadrawy
  • 15. Listeria monocytogenes mode of infection ? via CONTAMINATED FOOD,unpasteurised dairy products Vertical transmission➜ miscarriage Clinical picture of Listeria infection ? ❂ CONTAMINATED FOOD,unpasteurised dairy products ➜【meningitis, meningoencephalitis, or bacteremia】in IMMUNOSUPPRESSED PATIENTS, EXTREMES OF AGE (neonates and elderly) and PREGNANT. ♞ diarrhoea, flu-like illness ♞ pneumonia , meningoencephalitis ♞ ataxia and seizures ❂ blood cultures. CSF may reveal a pleocytosis, with 'tumbling motility' on wet mounts ❂ IV amoxicillin/ampicillin and gentamicin (resistant to cephalosporins) Dx of Listeria meningitis ? blood cultures. CSF may reveal a pleocytosis, with 'tumbling motility' on wet mounts Rx of Listeria meningitis ? Listeria meningitis should be treated with IV amoxicillin/ampicillin and gentamicin (resistant to cephalosporins) Clinical picture of Anthrax ? ❂ 【painless black eschar】 (cutaneous 'malignant pustule', but no pus) ❂ May cause marked edema ❂ Anthrax can cause gastrointestinal bleeding ✿ ciprofloxacin MRCP Infectious Disease Revision notes Dr.Sherif Elbadrawy
  • 16. Rx of Anthrax ? ✿ ciprofloxacin Who should be screened for MRSA? ✿ All elective admissions except (day patients➜ terminations of pregnancy & ophthalmic surgery. Patients admitted to mental health trusts) ✿ all emergency admissions How should a patient be screened for MRSA? Nasal swab and skin lesions or wounds Treatment of invasive MRSA infections in adults ? ✿ VANCOMYCIN ✿ DAPTOMYCIN ✿ TEICOPLANIN ✿ TIGECYCLINE ✿ QUINUPRISTIN & DALFOPRISTIN ✿ TELAVANCIN a semisynthetic lipoglycopeptide (-) cell wall synthesis ✿ ARBEKACIN a semisynthetic aminoglycoside antibiotic ✿ CEFTAROLINE, a fifth generation cephalosporin ✿ Adjunctive agents → Rifampin & Fusidic acid Etiology of Legionnaire's disease ? ✿ Legionella pneumophilia (intracellular bacteria) ✿ colonizes water tanks and hence questions may hint at air-conditioning systems or foreign holidays. Person-to- person transmission is not seen MRCP Infectious Disease Revision notes Dr.Sherif Elbadrawy
  • 17. Clinical picture of Legionnaire's disease ? ✿ Flu-like symptoms ✿ Dry cough ✿ Lymphopenia ✿ SIADH➜ Hyponatremia ✿ Deranged LFTs ✿ Urinary antigen ✿ Erythromycin Dx of Legionnaire's disease ? Urinary antigen Rx of Legionnaire's disease ? Erythromycin Etiology of Leptospirosis (Weil's disease) ? ❂ spirochaete Leptospira ➜spread by contact with infected rat urine.( ‫ا‬‫ﻟ‬‫ﻔ‬‫ﺎ‬‫ر‬ ‫ﺑ‬‫و‬‫ل‬ ) ❂ questions referring to sewage workers, farmers, vets or people who work in abattoir. Clinical picture of Leptospirosis ? ✿ Fever ✿ Flu-like symptoms ,no productive cough ✿ Jaundice ✿ (hepatorenal syndrome)(seen in 50% of patients) ✿ Weil's disease = Jaundice + Renal failure. ✿ Subconjunctival hemorrhage ✿ ±meningitis ❏ LP Dx meningitis ❏ High-dose 【benzylpenicillin or doxycycline】 MRCP Infectious Disease Revision notes Dr.Sherif Elbadrawy
  • 18. Rx of Leptospirosis ? ❏ Do LP 1st to Dx meningitis ❏ High-dose benzylpenicillin or doxycycline Etiology of Acute epiglottitis ? Hemophilus influenzae type B. Clinical picture of Acute epiglottitis ? ❊ Hemophilus influenzae type B. ❊ children bw 2 and 6 years. ✿ Rapid onset ✿ Unwell, toxic child ✿ Stridor ✿ Drooling of saliva ✿ Rapid Dx & Rx is essential to prevent airway obstruction Etiology of Lyme Disease ? ✰ Borrelia burgdorferi (USA) ✰ Borrelia afzelii (EUROPE) ✰ Borrelia garinii(EUROPE) Clinical picture of Lyme Disease ? 【Early features】 ☆ ERYTHEMA CHRONICUM MIGRANS (small papule often at site of the tick bite which develops into a larger annular lesion with central clearing, occurs in 70% of patients) ☆ Systemic symptoms: malaise, fever, arthralgia 【Later features】 ☆ CVS: heart block, myocarditis ☆ Neurological: cranial nerve palsies, meningitis ☆ Polyarthritis MRCP Infectious Disease Revision notes Dr.Sherif Elbadrawy
  • 19. Rx of Lyme Disease ? ☆【 Doxycycline】 (Should not be used for children < 8 years or pregnant or lactating women). ☆ ceftriaxone if disseminated disease ☆ Jarisch-Herxheimer reaction sometimes seen : fever, rash, tachycardia after first dose of antibiotic (> seen in syphilis, another spirochaetal disease) ➜ spontaneously settle after around four hours screening for TB ? Heaf test & Mantoux test (more accurate) ➜ to see if BCG is needed Interpretation of Mantoux test ? Erythema & induration > 10mm = positive result ➜ previous exposure including BCG Interpretation of Mantoux test ? (details) False negative Heaf & Mantoux tests in ? ✪ Miliary TB ✪ Sarcoidosis ✪ HIV ✪ Lymphoma ✪ Very young age (e.g. < 6 months) MRCP Infectious Disease Revision notes Dr.Sherif Elbadrawy
  • 20. Rx of active tuberculosis ? ❑ Initial phase - first 2 months (RIPE) ❁ Rifampicin ❁ Isoniazid ❁ Pyrazinamide ❁ Ethambutol ❑ Continuation phase - next 4 months ❁ Rifampicin ❁ Isoniazid Rx of Latent tuberculosis ? isoniazid alone for 6 months Meningeal tuberculosis ? prolonged period (at least 12 months) with the addition of steroids What's Directly observed therapy for TB ? ❑ 3 per week dosing regimen may be indicated in certain groups, including: ❁ Homeless people with active tuberculosis ❁ Patients who are likely to have poor concordance ❁ All prisoners with active or latent tuberculosis SEs of Anti-TB medications ? ☠ All of them cause HEPATOTOXICITY 〖 + 〗 ❏ Rifampin ➜ enzyme inducer [Revs up] , Red urine ❏ Isoniazid ➜ PERIPHERAL NEUROPATHY➜ Rx by PYRIDOXINE, Enzyme INHibitor [INH = INHibits] ❏ Pyrazinamide ➜HYPERURICEMIA ❏ Ethambutol ➜ EYE ➜Optic neuritis/color vision MRCP Infectious Disease Revision notes Dr.Sherif Elbadrawy
  • 21. Use of Streptomycin in TB ? aminogycoside used in resistant TB Rx of Asymptomatic patient with Hx of TB exposure and Heaf Test positive ? dual therapy (RI) for 5 months or INH for 6 months. Etiology of Leprosy ? Mycobacterium leprae Mycobacterium lepromatosis. Clinical picture of Leprosy ? Initially asymptomatic for 5 to 20 years. ➜ granulomas of the nerves, respiratory tract, skin (Nodular skin lesions), and eyes, hypopigmented skin lesions with reduced sensation. Dx of Leprosy ? Skin biopsy and needle test in cold area (ear lobule & elbow) ➜ Lepromatous leprosy ➜slowly growing.【ACID ALCOHOL FAST BACELLI】 tuberculoid leprosy ➜strong T cell immune response with no bacelli in the affected area skin biopsy. MRCP Infectious Disease Revision notes Dr.Sherif Elbadrawy
  • 22. Rx of Leprosy ? ☯ Pauci-bacillary leprosy (<5 lesions) rifampicin & dapsone for 6 months ☯ Multi-bacillary leprosy (>5 lesions) rifampicin, dapsone & clofazimine for 12 months Feature of severe malaria ? STAPH + Complications ▼ S➜ Schizonts on a blood film ▼ T➜ Temperature > 39 °c ▼ A➜ Anemia (severe) ▼ P➜ Parasitemia > 2% ▼ H➜ Hypoglycemia ▼ + ANY Complication Complications of malaria ? ▼ Brain➜ seizures, coma ▼ Lungs➜ ARDS ▼ Kidney➜ ARF: blackwater fever dt intravascular hemolysis ▼ Blood➜ DIC ▼ Pancreas➜ Hypoglycemia Rx of Uncomplicated falciparum malaria ? ✫ Strains resistant to chloroquine are prevalent in Asia and Africa ✫ artemisinin ➜ first-line therapy ❐ artemether plus lumefantrine, ❐ artesunate plus amodiaquine, or mefloquine,or sulfadoxine-pyrimethamine, ❐ dihydroartemisinin plus piperaquine ❐ Pregnant + malaria ➜ Quinine Rx of Severe falciparum malaria ? ❂ A parasite counts >2% IV Rx 【IV ARTESUNATE】 ❂ If parasite count > 10% ➜ exchange transfusion ❂ Shock may indicate coexistent bacterial septicemia - malaria rarely causes hemodynamic collapse MRCP Infectious Disease Revision notes Dr.Sherif Elbadrawy
  • 23. the most common cause of Non- falciparum malaria ? 【Plasmodium vivax】 ➜ Central America & India other causes ➜Plasmodium ovale & Plasmodium malariae➜Africa Rx of Non-falciparum malaria ? almost always chloroquine sensitive destroy liver hypnozoites ➜ by Primaquine (vivax & ovale) prophylaxis of malaria ? ⓵ Atovaquone + proguanil (Malarone) ➜ GI upset ⓶ Chloroquine ➜ CI in epilepsy ⓷ Doxycycline ➜ Photosensitivity, Oesophagitis ⓸ Mefloquine ➜ Dizziness, Neuropsychiatric disturbance, CI in epilepsy ✪ Pregnant ➜ chloroquine, proguanil + folate, Atovaquone + proguanil (Malarone) + folate ✪ children over 2 months ➜ diethyltoluamide (DEET) ✪ children over 12 years ➜ doxycycline Etiology of Leishmaniasis ? intracellular protozoa LEISHMANIA➜to humans by sand flies➜incubation period up to 10 years. Types of Leishmaniasis ? ✫ Cutaneous leishmaniasis ✫ Mucocutaneous leishmaniasis ✫ Visceral leishmaniasis (kala-azar) MRCP Infectious Disease Revision notes Dr.Sherif Elbadrawy
  • 24. Clinical picture of Cutaneous leishmaniasis ? ✫ Leishmania tropica or Leishmania mexicana ✫ Crusted lesion at site of bite➜ ulcer Clinical picture of Mucocutaneous leishmaniasis ? ✫ Leishmania brasiliensis ✫ Skin lesions➜ spread to involve mucosae of nose, pharynx Clinical picture of Visceral leishmaniasis (kala-azar) ? ✫ Leishmania donovani ✫ Mediterranean, Asia, South America, Africa ✫ Massive splenomegaly, hepatomegaly➜ Pancytopenia ✫ ↑ Fever,Typically ↓ appetite➜ weight loss (may be paradoxical ↑ appetite + weight loss ) Types of Trypanosomiasis ? African trypanosomiasis (sleeping sickness) American trypanosomiasis (Chagas' disease) Clinical picture of African trypanosomiasis(sleeping sickness)? ✱ Trypanosoma gambiense in West Africa ✱ Trypanosoma rhodesiense in East Africa. ➜Both spread by 【tsetse fly】. ❂ rhodesiense ➜acute course➜Trypanosoma chancre - tender SC nodule ➜post Cx lymphadenopathy➜ Meningoencephalitis MRCP Infectious Disease Revision notes Dr.Sherif Elbadrawy
  • 25. Rx of African trypanosomiasis(sleeping sickness)? Early disease: IV pentamidine or suramin Later disease or CNS involvement: IV melarsoprol Clinical picture of American trypanosomiasis (Chagas' disease) ? ✱ Trypanosoma cruzi. ❁ Acute Chagas' disease ➜95% asymptomatic ± CHAGOMA (an erythematous nodule at site of infection) + periorbital edema ❁ Chronic Chagas' disease:- ❅ HEART➜ Myocarditis ➜ HF & arrhythmias ➜ (leading cause of death) ❅ GIT ➜ megaesophagus ➜dysphagia & megacolon ➜ constipation. Rx of American trypanosomiasis (Chagas' disease) ? ❍ Benznidazole or nifurtimox ❍ Chronic Rx the complications e.g. heart failure Etiology of Schistosomiasis ? ♕ Schistosoma Hematobium➜ urinary schistosomiasis 【hematobium = hematuria】 Schistosomiasis is the most common cause of bladder calcification worldwide ♕ Schistosoma mansoni➜ intestinal schistosomiasis NB: Schistosoma mansoni & japonicum ➜ mesentric veins is the final distenation ➜ GIT symptoms. Clinical picture of Schistosoma Hematobium ? a 'swimmer's itch' in patients who have recently returned from Africa. Schistosoma Hematobium➜Frequency,Hematuria &Bladder calcification. a risk factor for squamous cell bladder cancer Single oral dose of 【PRAZIQUANTEL】 MRCP Infectious Disease Revision notes Dr.Sherif Elbadrawy
  • 26. Rx of Schistosoma Hematobium ? Single oral dose of 【PRAZIQUANTEL】 Clinical picture of Rabies ? Prodrome: headache, fever, agitation HYDROPHOBIA: water-provoking muscle spasms Hypersalivation What to do after an animal bite in at risk countries ? ❊ If immunised➜ 2 doses of vaccine ❊ If not immunised ➜ (HRIG) + full vaccination ❊ Cleanse wound ❊ CO-AMOXICLAV (DOCH) ❊ If penicillin-allergic ➜ doxycycline + metronidazole is recommended Etiology of Cat scratch disease ? Bartonella henselae Gram negative rod Clinical picture of Cat scratch disease ? ❍ Bartonella henselae Gram negative rod ❍ fever ,headache, malaise ❍ regional lymphadenopathy MRCP Infectious Disease Revision notes Dr.Sherif Elbadrawy
  • 27. Etiology of Chickenpox ? primary infection with VZV shingles is reactivation of dormant virus in dorsal root ganglion MOST CONTAGIOUS ORGANISM ? VZV Mode of transmission of VZV ? ✱ respiratory route ✱ Through shingles ✱ Infectivity = 4 days before rash, until 5 days after the rash(till all lesions had scabbed over) ✱ Incubation period = 11-21 days Clinical picture of VZV ? ✬ Fever initially ✬ Itchy, rash starting on head/trunk ➜ spread.1st macular then papular then vesicular ✬ Systemic upset is usually mild Rx of VZV ? ❂ Keep cool, trim nails ❂ Calamine lotion ❂ School exclusion:5 days from start of rash, until all lesions are crusted & dry. ❂ VZIG (for Immunocompromised patients and newborns with peripartum exposure)➜ If chickenpox develops ➜ IV aciclovir MRCP Infectious Disease Revision notes Dr.Sherif Elbadrawy
  • 28. complications of VZV ? ❂ Secondary bacterial infection of the lesions ❂ Pneumonia➜ MC & serious ➜ IV acyclovir ❂ Encephalitis ± cerebellar ❂ Disseminated hemorrhagic chickenpox Dx of VZV infection In pregnancy ? ❂ first step is to check antibodies ❂ both mother & fetus can be affected, a syndrome called fetal varicella syndrome ✺ not immune ➜ VZIG ASAP ✺ oral aciclovir if pregnant women with chickenpox present within 24 h of rash Clinical picture of FVS {Fetal varicella syndrome} ? ✯ before 20 weeks ✯ skin scarring,microphthalmia, microcephaly & learning disabilities, limb hypoplasia, Rx of VZV infection In pregnancy ? ✺ not immune to varicella ➜ VZIG ASAP ✺ oral aciclovir if pregnant women with chickenpox present within 24 h of rash Etiology of Measles ? ✺ RNA paramyxovirus ✺ Spread by droplets ✺ Infective from prodrome until 5 days after rash starts MRCP Infectious Disease Revision notes Dr.Sherif Elbadrawy
  • 29. Clinical picture of Measles ? ✺ Prodrome: irritable, conjunctivitis, fever ✺ Koplik spots (before rash): white spots ('grain of salt') on buccal mucosa ✺ Rash: starts behind ears then to whole body, discrete maculopapular rash becoming blotchy & confluent Complications of Measles ? [NB:Pancreatitis and infertility may follow mumps infection] ✺ Encephalitis: 1-2 weeks after the onset of the illness. ✺ Febrile convulsions ✺ Pneumonia, tracheitis ✺ Keratoconjunctivitis, corneal ulceration ✺ Diarrhoea ✺ ↑ incidence of appendicitis ✺ Myocarditis ✺ Subacute sclerosing panencephalitis: very rare, may present 5-10 years following the illness Management of contacts of Measles ? ✺ a child not immunized + contact with measles then MMR (vaccine- induced measles antibody > rapid than natural infection) ✺ Use within 72 hours Etiology of Gonorrhoea ? Gram negative diplococcus Neisseria gonorrhoea Acute infection can occur on any MM surface, typically genitourinary but also rectum and pharynx Clinical picture of Gonorrhoea ? ✺ ♂ urethral discharge, dysuria ✺ ♀ cervicitis ➜ vaginal discharge ✺ Rectal and pharyngeal infection is usually asymptomatic, but may present as rectal bleeding ✺ Local complications➜ urethral strictures, epididymitis and salpingitis ➜ (infertility). Disseminated infection may occur MRCP Infectious Disease Revision notes Dr.Sherif Elbadrawy
  • 30. Rx of Gonorrhoea ? ✺ Cephalosporins ("Oral" Cefixime or Ceftriaxone "single dose IM") ➜ DOC ✺ Ciprofloxacin is no more DOC dt ↑ resistance to ciprofloxacin Etiology of Disseminated gonococcal infection ? ✩ Hematogenous spread from asymptomatic genital infection. ✩ Triad : 【tenosynovitis, migratory polyarthritis and dermatitis】 (maculopapular or vesicular) ✩ Later complications ➜ septic arthritis(MCC of septic arthritis in young adults), endocarditis and perihepatitis Etiology of Genital warts ? (condylomata accuminata) human papilloma virus HPV especially types 6 & 11 Which types of human papilloma virus predispose to cervical cancer ? (16, 18 & 33) Clinical picture of Genital warts ? (condylomata accuminata) HPV, Small (2 - 5 mm) fleshy protuberances on the genital regions + slightly pigmented bleeding or itching. MRCP Infectious Disease Revision notes Dr.Sherif Elbadrawy
  • 31. Rx of Genital warts ? (condylomata accuminata) ★ Topical podophyllum or cryotherapy➜ first-line ★ Multiple, non-keratinised warts ➜ Topical podophyllum ★ solitary keratinised warts ➜ cryotherapy, Imiquimod ★ recurrence is common ★ HPV clear without intervention within 1-2 years Etiology of Genital Ulcers ? Syphilis➜Painless Lymphogranuloma venereum➜Painless Granuloma inguinale➜Painless Herpes Simplex Virus➜Painful Chancroid➜Painful Dx of Syphilis ? Treponema pallidum is a very sensitive organism can't grow on artificial media. The diagnosis is therefore usually based on clinical features; serological tests and microscopic examination of infected tissue Serological tests for Dx of Syphilis ? ✾ Cardiolipin tests (not treponeme specific) ✷ VDRL & RPR ✷ Insensitive in late syphilis ✷ Becomes negative after treatment ✾ Treponemal specific antibody tests ✷ TPHA ✷ Remains positive after treatment Causes of false positive cardiolipin tests ❂ pregnancy ❂ SLE, anti-phospholipid syndrome ❂ TB ❂ leprosy ❂ malaria ❂ HIV MRCP Infectious Disease Revision notes Dr.Sherif Elbadrawy
  • 32. Etiology of Lymphogranuloma venereum ? Chlamydia trachomatis. Clinical picture of Lymphogranuloma venereum ? Chlamydia trachomatis. ✾ Stage 1: painless pustule ➜ ulcerate ✾ Stage 2: painful inguinal lymphadenopathy ✾ Stage 3: proctocolitis Types of Herpes Simplex Virus ? HSV-1 and HSV-2 it was thought HSV-1 ➜ oral lesions (cold sores) & HSV-2 ➜genital herpes it is now known there is considerable overlap Clinical picture of Herpes Simplex Virus ? ❂ Cold sores ❂ Painful genital ulceration ❂ Primary infection ➜ ± gingivostomatitis Rx of Herpes Simplex Virus ? ❂ Gingivostomatitis: oral aciclovir, chlorhexidine MW ❂ Cold sores: topical aciclovir ❂ Genital herpes: oral aciclovir, ↑exacerbations ➜ longer term aciclovir MRCP Infectious Disease Revision notes Dr.Sherif Elbadrawy
  • 33. Etiology of Chancroid ? Hemophilus ducreyi Clinical picture of Chancroid ? ❃ Hemophilus ducreyi ❃ painful genital ulcers ❃ inguinal lymphadenopathy. ❃ Grey or yellow purulent exudate⇔ Syphilis is Non-exudative ❃ Soft edge ⇔ Syphilis is Hard (indurated) edge Common causes of Vaginal discharge ? ✯ Physiological ✯ Candida➜'Cottage cheese' discharge Vulvitis Itch ✯ Trichomonas vaginalis➜ Offensive, yellow/green, frothy discharge, Vulvovaginitis Strawberry cervix ✯ Bacterial vaginosis ➜Offensive, thin, white/grey, 'FISHY' discharge Less common causes of Vaginal discharge ? ✯ cervical infections such as Chlamydia and Gonorrhoea ➜rarely the presenting symptoms ✯ Ectropion ✯ Foreign body ✯ Cervical cancer Etiology of Bacterial vaginosis (BV) ? ✯ overgrowth of anaerobic organisms➜ 【Gardnerella vaginalis】 ➜ ↓ lactobacilli ➜↓ Lactic acid ➜↑ vaginal pH. not a STD but seen almost exclusively in sexually active women. ✯ Asymptomatic in 50% MRCP Infectious Disease Revision notes Dr.Sherif Elbadrawy
  • 34. Amsel's criteria for diagnosis of Bacterial vaginosis (BV) ? 【Gardnerella vaginalis】 ❃ 3 of the following 4 points ✴ Thin, white homogenous discharge ✴ Clue cells on microscopy ✴ Vaginal pH > 4.5 ✴ Positive whiff test (addition of potassium hydroxide ➜ fishy odour) Complications of Bacterial vaginosis in pregnancy ? ✯ preterm labour ✯ low birth weight ✯ chorioamnionitis ✯ late miscarriage ✯ oral metronidazole is used throughout pregnancy. Rx of Bacterial vaginosis (BV) ? ✯ Oral metronidazole for 5-7 days ✯ 70-80% initial cure rate ✯ Relapse rate > 50% within 3 months most common sexually transmitted infection in the UK ? ✯ Chlamydia ✯ caused by Chlamydia trachomatis, an obligate intracellular pathogen ✯ ≅ 10 % in UK women. Clinical picture of Genital Chlamydia infection ? Asymptomatic in around 70% of women Women: cervicitis (discharge, bleeding), dysuria Men: urethral discharge, dysuria MRCP Infectious Disease Revision notes Dr.Sherif Elbadrawy
  • 35. complications of Genital Chlamydia infection ? ✯ Epididymitis ✯ Pelvic inflammatory disease ✯ Endometritis ➜ 【1st 3 causes➜ Infertility】 ✯ ↑ incidence of ectopic pregnancies ✯ Reactive arthritis ✯ Perihepatitis (Fitz-Hugh-Curtis syndrome) Dx of Genital Chlamydia infection ? ✯ no Traditional cell culture ✯ Ix of choice➜Nuclear Acid Amplification Tests (NAATs)➜Use Urine (first void urine sample), vulvovaginal swab or cervical swab ✯ NB: Neisseria Gonorrhoea ➜ Gram stain➜ Gram - ve diplococci ➜ If the swab showed non-specific urethritis ➜ Chlamydia is most likely. Both many times infect together Screening of Genital Chlamydia infection ? screening all sexually active patients aged 15-24 years Rx of Genital Chlamydia infection ? ✿ 【 Azithromycin 】〖first-line〗➜single dose➜better compliance ✿ Doxycycline 7 day course ➜may be used but Azithromycin is better dt better compliance. ✿ pregnant➜ erythromycin or amoxicillin ✿ in ♂ with symptomatic infection partner notification➜ (all partners from 4 weeks prior to symptoms) ✿ in ♀& asymptomatic ♂ partner notification ➜ ( last 6 months or the most recent sexual partner) ✿ partners of confirmed Chlamydia Pt ➜treat then test ✿ A test of cure following treatment. Clinical picture of Chlamydia psittaci (psittacosis)(parrot disease, parrot fever) ? malaise, fever, myalgias and pneumonia Exposure to an ill bird and a rash (Horder's spots) are pathognomonic. MRCP Infectious Disease Revision notes Dr.Sherif Elbadrawy
  • 36. Rx of Chlamydia psittaci (psittacosis) ? Erythromycin or tetracyclines are the drugs of choice. Rx of Asymptomatic bacteruria in pregnant women ? amoxicillin or cephalosporin Rx of Lower urinary tract infections in women (cystitis) ? trimethoprim or nitrofurantoin for 3 days Rx of acute lower urinary tract infections in pregnant women ? amoxicillin or an oral cephalosporin for 7 days Rx of acute pyelonephritis ? ✼ hospital admission ✼ a broad-spectrum cephalosporin or a quinolone for 10-14 days ✼ ciprofloxacin for 7 days or co- amoxiclav for 14 days MRCP Infectious Disease Revision notes Dr.Sherif Elbadrawy
  • 37. most common congenital infection in the UK ? Cytomegalovirus 〚Maternal infection is usually asymptomatic〛 congenital Rubella infection ? PDA = (P)atent DA, (D)eafness, (A)nterior chamber 【CATARACTS & Glaucoma】 ➜ Anterior chamber 【heart disease (PDA)】 【Sensorineural deafness】 congenital Toxoplasma infection ? 【CEREBRAL CALCIFICATION➜ Hydrocephalus】 【CHORIORETINITIS】 ➜ Posterior chamber congenital CMV infection ? 【Growth retardation】 【Purpuric skin lesions】 Less diagnostic features of congenital Toxoplasmosis, Rubella & CMV ? 〖All of them〗➜ Hepatosplenomegaly,Cerebral palsy 【Toxoplasmosis】➜ Anemia 【Rubella】➜ Purpuric skin lesions 'Salt and pepper' ,chorioretinitis Microphthalmia ,Growth retardation 【CMV】➜ Anemia, Sensorineural deafness, Encephalitiis ,Pneumonitis, Jaundice MRCP Infectious Disease Revision notes Dr.Sherif Elbadrawy
  • 38. Etiology of Toxoplasmosis ? Toxoplasma Gondii➜ a protozoa➜ infection via GIT, lung or broken skin➜the oocysts release trophozoites ➜migrate widely around the body including to the eye, brain and muscle. The usual animal reservoir is the CAT ± rats may carry the disease. Clinical picture of Toxoplasmosis ? ✾ mostly asymptomatic ✾ If symptomatic➜ self-limiting infection ✾ same as infectious mononucleosis (fever, malaise, and lymphadenopathy). ✾ meningioencephalitis and myocarditis. (< common) Dx of Toxoplasmosis ? Antibody test Sabin-Feldman dye test Rx of Toxoplasmosis ? Pyrimethamine + Sulphadiazine for at least 6 weeks. Mnemonic for Infectious Causes of Bloody Diarrhea ? CHESS Campylobacter Hemorrhagic E. coli (O157:H7) Entamoeba histolytica Salmonella Shigella MRCP Infectious Disease Revision notes Dr.Sherif Elbadrawy
  • 39. The most common cause of Travellers' diarrhea ? Escherichia coli at least 3 loose to watery stools in 24 h ± Abdominal pain, fever, nausea, vomiting or blood in the stool most common infection from reheated rice ? Bacillus cereus infection Etiology of 'acute food poisoning' ? Staphylococcus aureus, Bacillus cereus Clostridium perfringens. sudden onset of nausea, vomiting and diarrhea after the ingestion of a toxin. Clinical picture of Cholera diarrhea ? Vibro cholerae - Gram negative bacteria Profuse rice watery diarrhea Severe dehydration ➜ weight loss Not common amongst travellers Clinical picture of Campylobacter diarrhea ? ✯ A flu-like prodrome ➜ abdominal pains➜ Fever and diarrhoea ± blood ✯ may be complicated by Guillain-Barre syndrome MRCP Infectious Disease Revision notes Dr.Sherif Elbadrawy
  • 40. Campylobacter Rx ? ✯ usually self-limiting ✯ immunocompromised ➜ antibiotics if severe symptoms [bloody diarrhoea, > 8 stools / day] ➜ clarithromycin Clinical picture of Amoebiasis diarrhea ? Gradual onset bloody diarrhea Abdominal pain and tenderness May last for several weeks Differnt Incubation periods of Diarrhea ? ✯ 1-6 hrs: Staphylococcus aureus, Bacillus cereus ✯ 12-48 hrs: Salmonella, Escherichia coli ✯ 48-72 hrs: Shigella, Campylobacter ✯ > 7 days: Giardiasis, Amoebiasis Rx of Cholera diarrhea ? ✯ Oral rehydration therapy ✯ doxycycline, ciprofloxacin Etiology of Giardiasis ? flagellate protozoan Giardia lamblia. It is spread by the faeco-oral route MRCP Infectious Disease Revision notes Dr.Sherif Elbadrawy
  • 41. Clinical picture of Giardiasis ? ✯ Often asymptomatic ✯ Lethargy, bloating, abdominal pain ✯ Non-bloody diarrhoea ✯ Chronic/prolonged diarrhoea, malabsorption and lactose intolerance Dx of Giardiasis ? Stool microscopy for trophozoite and cysts are CLASSICALLY NEGATIVE ➜ duodenal fluid aspirates or 'string tests' (fluid absorbed onto swallowed string) are sometimes needed. Rx of Giardiasis ? Metronidazole Loa Loa CP ▩ CP ➜ pruritus, urticaria, Calabar swellings: transient, swelling of soft-tissue around joints, 'eye worm' - the dramatic presentation of subconjuctival migration of the adult worm. ♕ Rx ➜ diethylcarbamazine (DEC), high loa loa microfilaraemia is associated with encephalopathy following Rx with either Ivermectin or DEC. dt death of vast numbers of blood microfilaria. Both of these drugs are CI if loa loa microfilaraemia > 2500 mf/ml. Most common E -coli infections in humans ? ✯ Diarrhoeal illnesse ✯ UTIs ✯ Neonatal meningitis MRCP Infectious Disease Revision notes Dr.Sherif Elbadrawy
  • 42. Clinical picture of EHEC (E coli O157:H7) infection ? ✯ Diahrrea ✯ Renal Failure or Impairment ✯ Thrombocytopenia ✯ hemolytic uremic syndrome ✯ hemorrhage➜ ↓ Hb hemolytic uremic syndrome can be caused by which bacteria ? EHEC (E coli O157:H7) infection spread by contaminated ground beef. Which disease Can Body louse transmit ? ✯ Body lice can spread epidemic typhus, trench fever, and louse- borne relapsing fever. ✯ louse-borne (epidemic) typhus is no longer widespread, ✯ outbreaks of this disease still occur during overcrowedness with low sanitation:- ◌ times of war, ◌ civil unrest, ◌ natural or man-made disasters, ◌ in prisons ◌ chronic poverty Clinical picture of Salmonella infection (Typhoid) ? ❂ headache,fever, and arthralgia ❂ Relative bradycardia ❂ Abdominal pain, distension ❂ Constipation: > in typhoid ❂ Rose spots: > in paratyphoid complications of Salmonella infection (Typhoid) ? ❂ Osteomyelitis (Salmonella is MCC in sickle cell disease Osteomyelitis ) ❂ GI bleed/perforation ❂ Meningitis ❂ chronic carriage (1%, more likely if adult females) MRCP Infectious Disease Revision notes Dr.Sherif Elbadrawy
  • 43. Types of Shigella bacterial diarrhea ? S sonnei (e.g. from UK) may be mild, S flexneri or S dysenteriae from abroad may cause severe disease Rx of Shigella bacterial diarrhea ? Ciprofloxacin Definition of Pyrexia of Unkown Origin ? a fever of > 3 weeks which undiagnosed after a week in hospital Etiology of Pyrexia of Unkown Origin ?《 Infections Can Make Undergraduates Die》 ? 〖Infection (20-40%)〗 ❂ TB ❂ endocarditis ❂ EBV ❂ CMV 〖Connective tissue disorders (20%)〗 ❂ RA ❂ SLE ❂ Adult-onset Still's disease ❂ Temporal arteritis 〖Malignancy (10-20%)〗 ❂ Lymphoma ❂ Leukaemia ❂ HCC ❂ RCC ❂ Atrial myxoma 〖Undiagnosed (20%)〗 〖Drugs〗phenytoin Clinical picture of African Tick Typhus ? dt Rickettsiae conorii BLACK SPOTS ON THIGH Hx of tick bites Low grade fever Faint macular rash MRCP Infectious Disease Revision notes Dr.Sherif Elbadrawy
  • 44. Etiology of Rocky Mountain Spotted Fever ? Rickettsia,spreads by ticks, common in USA. Clinical picture of Rocky Mountain Spotted Fever ? ❀ Rickettsia,spreads by ticks, common in USA. ❀ Fever ❀ Rash on hands, feet ➜ peel ❀ Tachycardia with no hypotension (unlike Staphy Toxic Shock Syndrome) ❀ Rx Doxycycline Rx of Rocky Mountain Spotted Fever ?(Rickettsia) Doxycycline Etiology of Mediterranean Spotted Fever ?(Boutonneuse fever) Rickettsia conorii and transmitted by the dog tick Rhipicephalus sanguineus Clinical picture of Mediterranean Spotted Fever ?(Boutonneuse fever) ↑ FEVERS, MYALGIA and joints pain, severe headache, photophobia and diarrhea. bite site ➜ black spots or ulcerous crust (tache noire).4th day ➜ widespread rash appears, first macular ➜ maculopapular ± petechial. MRCP Infectious Disease Revision notes Dr.Sherif Elbadrawy
  • 45. Rx of Mediterranean Spotted Fever ? Doxycycline. Etiology of viral hemorrhagic fever ? ❂ Dengue fever ❂ yellow fever ❂ Lassa fever ❂ Ebola Given the current outbreak in West Africa, this group of infections must be considered as a differential in returning travellers presenting with a fever. Etiology of Dengue fever ? dengue virus Transmitted by the Aedes aegypti mosquito Clinical picture of Dengue fever ? ❀ Dengue virus (Aedes aegypti mosquito) ❀ incubation period of 7 days ❀ Headache (often retro-orbital) ❀ Facial flushing (dengue) ❀ Maculopapular rash (resembling measles) ❀ Thrombocytopenia &↑ LFTs ❀ DIC ❀ Rx ➜ symptomatic ➜ fluid resuscitation, blood transfusion Rx of Dengue fever ? symptomatic ➜fluid resuscitation, blood transfusion MRCP Infectious Disease Revision notes Dr.Sherif Elbadrawy
  • 46. Etiology of Infectious mononucleosis (glandular fever) ? ❀ Epstein-Barr virus ( HHV-4). ❀ adolescents and young adults. Clinical picture of Infectious mononucleosis (glandular fever) ? ❀ Sore throat,Fever.Palatal petechiae ❀ Lymphadenopathy ❀ Splenomegaly in 50% of patients ➜ ± splenic rupture ❀ Hepatitis ❀ Malaise, anorexia, headache ❀ lymphocytosis ❀ Hemolytic anaemia Rx of Infectious mononucleosis (glandular fever) ? ❀ symptomatic ➜fluid resuscitation,Rest during the early stages,avoid alcohol ❀ analgesics ❀ avoid contact sports for 8 weeks ➜↓risk of splenic rupture Malignancies associated with EBV infection ? Burkitt's lymphoma Hodgkin's lymphoma Nasopharyngeal carcinoma Criteria Hepatitis E infection ? RNA virus faecal-oral route Common in Asia, Africa & Mexico significant mortality (about 20%) during pregnancy Does not cause chronic disease no vaccine MRCP Infectious Disease Revision notes Dr.Sherif Elbadrawy
  • 47. Etiology of Swine Flu ? influenza A virus and the most common cause of flu in humans. The 2009 pandemic was caused by a new strain of the H1N1 virus. Pts at risk of swine flu ? Patients with chronic Disease + Pts on immunosuppressants Pregnant women Young children < 5 years old Clinical picture of swine flu ? Fever > 38ºC Myalgia Lethargy Headache Rhinitis Sore throat Cough Diarrhoea and vomiting may be complicated by ARDS. Rx of swine flu ? (H1N1) (ARDS) ❀ Oseltamivir (Tamiflu):- Oral medication, A neuraminidase inhibitor SEs ➜ NVD & headaches ❀ Zanamivir (Relenza) Inhaled or IV for acutely ill pts. A neuraminidase inhibitor SEs ➜ bronchospasm in asthmatics Etiology of Erythema infectiosum (fifth disease or 'slapped-cheek syndrome') ? Parvovirus B19 MRCP Infectious Disease Revision notes Dr.Sherif Elbadrawy
  • 48. Clinical picture of Erythema infectiosum (fifth disease or 'slapped- cheek syndrome') ? ❂ Parvovirus B19 ❂ may be Asymptomatic ❂ lethargy, fever, headache ❂ 'slapped-cheek' rash spreading to proximal arms and extensor surfaces ❂ Pancytopenia in immunosuppressed ❂ Aplastic crises in sickle-cell disease Etiology of Orf ? parapox virus a condition found in sheep and goats although it can be transmitted to humans. Clinical picture of Orf ? ❂ parapox virus ❂ a condition found in sheep and goats ❂ small, raised,papules ( red-blue)in the hands and arms ➜↑ in size,become flat- topped and hemorrhagic Most common cause of cutaneous larva migrans ? ancylostoma braziliense Common in Central and Southern America Strongyloides stercoralis causes what ? ✾ larva currens - similar appearance to cutaneous larva migrans but moves through the skin at a far greater rate ✾ Acquired percutaneously (e.g. Walking barefoot) ✾ EOSINOPHILIA ✾ pruritus ✾ Abdominal pain, diarrhoea, pneumonitis ✾ May cause gram negative septicemia due to carrying of bacteria into bloodstream MRCP Infectious Disease Revision notes Dr.Sherif Elbadrawy
  • 49. Rx of Strongyloides stercoralis ? thiabendazole, albendazole. Ivermectin also used, particularly in chronic infections. Commonest cause of visceral larva migrans ? TOXOCARA CANIS dt ingesting eggs from soil contaminated by dog faeces eye granulomas, liver/lung involvement Etiology of Cysticercosis ? Taenia solium (from pork) and Taenia saginata (from beef). Rx niclosamide Rx of Cysticercosis ? niclosamide Etiology of Hydatid disease ? dog tapeworm Echinococcus granulosus Often seen in farmers May cause liver cysts Rx Albendazole MRCP Infectious Disease Revision notes Dr.Sherif Elbadrawy
  • 50. Rx of Hydatid disease ? Albendazole The most common infecting organism after a dog bite ? Anaerobic mouth flora pasteurella multocida capnocytophaga & strept pyogenes may be present Commonest cause of viral gastroenteritis ? ✬ Norwalk virus (norovirus) ➜Watery diarrhea ✬ sporadic & epidemic diarrhea ✬ Dx by ELISA ➜ viral Ag detection ✬ (other viral causes of watery diarrhea include Rotavirus and Adenovirus) The rapid development of focal neurological signs, in the context of severe immunosuppression with HIV and the presence of multiple ring-enhancing lesions in the brain ? ✬ toxoplasmosis Tuberculosis, progressive multifocal leucoencephalopathy (hyperintense signal on T2-weighted images,cerebellar), lymphoma a (solitary lesion) & HIV encephalopathy are also possible diagnoses in the present case. Definition of Tropical sprue ? a malabsorption disease commonly found in the tropical regions, marked with abnormal flattening of the villi and inflammation of the lining of the small intestine. MRCP Infectious Disease Revision notes Dr.Sherif Elbadrawy
  • 51. pigment producing bacteria: "ISRAEL has YELLOW SAND" ? actinomyces israeli - yellow "sulfur" granules primary affect face & neck➜usually present as abscess. HIV-positive. Her CD4 count is > 200 and she presented with salmonella gastroenteritis. Rx successfully. In addition to (HAART), we should give prophylaxis against which one of the following organisms ? Toxoplasma gondii Cytomegalovirus Pneumocystis jiroveci Salmonellae Clyptococci ✬ SALMONELLAE ✬ prophylaxis for (P. jiroveci, cytomegalovirus, Toxopiasma and cryptococcal infections) can be D/C in patients with CD4 > 200. Long-term therapy ➜to prevent a recurrence in people diagnosed with salmonellosis. ➜ despite HAART, patients infected with HIV appear to mount an ineffective antibody response to salmonella infection. ✬ Ciprofloxacin is usually the first choice Which one of the following options is true with respect to prophylaxis against pneumococcal infection in post splenectomy pt.? ☆ Penicillin is not indicated ☆ Penicillin prophylaxis 500 mg bd is indicated for at least a 2- year period ☆ Penicillin prophylaxis 250 mg bd is indicated for a 2-year period ☆ They should be immediately revaccinated against pneumococcus ☆ Pneumococcal vaccination should be repeated every 10 years ☆ Penicillin prophylaxis 500 mg bd is indicated for at least a 2-year period !! ☆ Pneumococcal vaccination should be repeated every 5 years risk of developing various infections and oppertunistic diseases at different CD4 counts landmarks in HIV infected patients ? ☆ CD4 count above 700 ➜Normal ☆ CD4 count 200-500 ➜ Oral thrush, Kaposi sarcoma, TB, Zoster, and lymphoma ☆ CD4 count 100-200 ➜ PCP pneumonia, PML, Dementia, Histoplasmosis, and Coccidiomycosis ☆ CD4 count 50-100 ➜ Toxoplasmosis, Cryptococcosis, and Cryptosporodiosis ☆ CD4 count less than 50 ➜ CMV, Mycobacterium avium complex, CNS lymphoma characteristic diseases in AIDS patients ? ☆ Hairy Leukoplakia ☆ Kaposi Sarcoma (CD Count: 200 - 500 cells/ml) ☆ High-grade B-Cell Lymphomas ☆ Cervical cancer ☆ Wasting syndrome ☆ Nephropathy ☆ Dementia (CD Count: 100 - 200 cells/ml) MRCP Infectious Disease Revision notes Dr.Sherif Elbadrawy
  • 52. AIDS Oppurtunistic Infections ? ✪ LUNG - ✷ PCP - pneumonia, also in bone marrow (CD100 - 200 ) ✷ Mycobacterium TB - (CD : 200 - 500) ✷ Mycobacterium avium-intracellulare - also in GI tract (CD: <50) ✷ Coccidioidomycosis ✷ Histoplasmosis ✷ CMV - also affect retina, adrenals and GI tract (CD: <50) ✷ Cryptococcus ✪ Esophagus - ✷ HSV - also cause CNS encephalitis (CD : 200 - 500) ✷ Candida - oral thrush in oral pharynx (CD: 200 - 500) ✪ GI Tract - ✷ Giardia Lamblia ✷ Cryptosporidium ✪ CNS - ✷ Aspergillus - spores can also affect lungs and blood vessels ✷ Toxoplasmosis - MC opportunistic infection of brain (CD : <100) ✷ Cryptococcus -meningitis(most common presentation for cryptococcus "also in the lung") (chronic only, not acute) (CD :<100 ) ✷ JC virus - progressive multifocal leukoencephalopathy (CD: <50) When should the patient in a pre splenectomy case receive conjugate pneumococcal vaccination? ✬ 1 week before operation ✬ 5 days before operation ✬ 2 weeks before operation ✬ 1 month after operation ✬ Postoperatively 2 weeks before operation also Hib & meningococal vaccine A throat swab reveals diphtheria Which one of the following options is the next most appropriate action? ✬ Examine the cerebrospinal fluid (CSF) ✬ Blood cultures ✬ Ceftriaxone ✬ Antitoxin ✬ Hydrocortisone Antitoxin Clinical picture of Herpes simplex virus kerariris ? - Herpes simplex virus (HSV) keratitis is characterised by the acute onset of pain, blurred vision, conjunctival injection and DENDRITIC ULCERATION of the cornea - HSV keratitis can cause corneal blindness and its treatment is urgent Which one of the following options is the most common side-effect of pyrazinamide? Hepatitic dysfunction Hyperuricaemia Colour vision changes Dizziness Neurotoxicity Hepatitic dysfunction Hyperuricaemia is a characteristic SE but not the most common. MRCP Infectious Disease Revision notes Dr.Sherif Elbadrawy
  • 53. Clinical picture of chickenpox ? highly contagious disease caused by primary infection with varicella zoster virus (VZV) prodrome of constitutional symptoms ➜ characteristic vesicular rash all over the body and/or oral sores ➜ blister stage, intense itching is usually present. Blisters may also occur on the palms, soles, and genital area. ± oral cavity & tonsil areas in the form of small ulcers which can be painful or itchy or both Can Interferon administration in the acute phase of infection prevents the development of the chronic hepatitis B carrier state ? 【NO】 Interferon administration in the acute phase of infection has not shown any benifit.but it's effective in the Rx of chronic hepatitis B. Etiology of Yellow fever ? flavivirus, and can vary in severity from a mild illness to the severe classical form. present in TROPICAL areas➜ Africa and South America ➜ transmitted to human by mosquito. Clinical picture of Yellow fever ? ⌘ Aedes mosquitos ⌘ incubation period = 2 - 14 days ⌘ severe flu-like illness with fever up to 40 ± epigastric pain and vomiting ⌘ Relative bradycardia ⌘ a recovery phase and the patient feels well for several days ⌘ severe fever again+ jaundiced, hepatomegaly, ➜ haematemesis, bleeding gums ➜ DIC. ⌘ Councilman bodies (inclusion bodies) may be seen in the hepatocytes Rx of Yellow fever ? Supportive therapy only Mortality up to 40% MRCP Infectious Disease Revision notes Dr.Sherif Elbadrawy
  • 54. Dx of malaria ? ❂ Three separate blood films should be sent to r/o malaria when it is suspected. ❂ Thick & thin film blood smear ➜ film is +ve ➜ send the other blood samples.. if the pt is severely ill or deteriorating ➜ start antimalarial Rx ASAP before C&S result. ❂ Although the patient may use malaria prophylaxis, they're still at risk of malaria infection Clinical picture of Cerebral toxoplasmosis in AIDS ? ❊ most common infection of the CNS in AIDS when the CD4 < 200 ❊ focal neurological disturbance, headache, confusion, fever and convulsions Dx of Cerebral toxoplasmosis in AIDS ? Toxoplasma cysts in the brain➜ multiple ring-enhancing masses with surrounding oedema Rx of Cerebral toxoplasmosis in AIDS ? Pyrimethamine — an antimalarial medication Sulfadiazine — an antibiotic used in combination with pyrimethamine to treat toxoplasmosis + Folic acid What Is the Difference between HIV and AIDS? When a person is infected with the human immunodeficiency virus (HIV) we say that he or she is "HIV positive" .A person who has HIV is classified as having AIDS if one of two things happens: ❊ if the CD4 count has dropped below 200/cc, or ❊ if an HIV-related infection or HIV-related cancer develops. A patient may have 200 CD4 cells or less and feel very healthy, but he or she still has AIDS by definition. MRCP Infectious Disease Revision notes Dr.Sherif Elbadrawy
  • 55. F Pt. returns from South Africa with confusion and headache, no neck stiffness. She has a purpuric rash. Malaria prophylaxis included mefloquine.most important Ix ? Lumbar puncture Computed tomography (CT) scan Multiple blood cultures Malaria films Clotting Malaria films Although the patient in the present case has used malaria prophylaxis, she is still at risk of malaria infection Bacterial endocarditis is treated with penicillin. What single additional step is undertaken to prevent penicillin resistance? Double the penicillin dose Give a suitable B-lactam combination Add macrolides Add aminoglycosides Interrupt penicillin treatment Add aminoglycosides Homosexual male with a lesion on his penis that was initially nodular and painless➜ a heaped-up ulcer. Sampling ➜Biopsy: large infected mononuclear cells containing many Donovan bodies. What diagnosis fits best with this clinical picture? Penile carcinoma Lymphogranuloma venereum Chancroid Genital herpes Granuloma inguinaie Granuloma inguinaie Caused by➜ Klebsiella granulomatis key features➜ painless indurated nodule➜heaped- up ulcer + infected mononuclear cells containing many Donovan bodies Rx➜ tetracycline or ampicillin 25 y woman gives birth at full term to an otherwise well baby with UNILATERAL MICROPHTHALMIA. She recalls a rash during the first trimester of her pregnancy, but cannot remember any other details of the rash. Which one of the following options is the most likely causative agent? ☆ Syphilis ☆ Varicella zoster virus (VZV) ☆ Rubella ☆ Cytomegalovirus (CMV) ☆ Parvovirus B19 ✱ Varicella zoster virus (VZV) ☆ The fetus develops episodes of shingles that affect development of the involved dermatome ☆ Congenital syphilis, CMV and rubella tend to cause symmetrical problems Etiology of mumps ? Paramyxovirus MRCP Infectious Disease Revision notes Dr.Sherif Elbadrawy
  • 56. Etiology of Q fever ? Coxiella burnetii obligate intracellular bacteria & doesn't grow on standard culture media. man infection by inhalation or drinking unpasteurized milk. Criteria of Mycobacterium avium complex (MAC) infection ? ✬ opportunist mycobacterium found in soil, water, dust, milk ✬ usually attacks patients with pre-existing lung disease ➜ COPD , bronchiectasis or immunosuppressed ✬ Dx➜ X-ray ➜cavitation,sputum C&S ✬ Rx➜ rifampicin and ethambutol for 24 months ± lsoniazid . ✬ if Rx failure ➜ add Ciprofloxacin or clarithromycin ✬ surgical resection of the disease area is an option in fit pts. Etiology of Paragonimiasis ? Paragonimus westermani is in the genus of flatworms,( an infectious lung fluke). lower lobe cavitating lesions, an eosinophilia A granulomatous response to the eggs may occur DD from TB ➜lack of systemic symptoms (fever, weight loss),-ve acid-fast bacilli most common cause of failure of antiretroviral Rx ? Poor compliance to Rx. Human T-cell lymphotrophic virus type I is associated with which type of cancer ? acute T-cell lymphoma/leukaemia MRCP Infectious Disease Revision notes Dr.Sherif Elbadrawy
  • 57. Human papilloma virus is associated with which type of cancer ? carcinoma of anal canal in homosexual men A 7-y boy who has come from lndia ➜ mild fever (37.5 °C), maculopapular rash and pharyngitis. difficulty swallowing, even his own saliva+ greyish membrane surrounding the tonsils+ regional lymphadenopathy. most likely diagnosis? ◆ Infectious mononucleosis ◆ Diphtheria ◆ Rubella ◆ Measles ◆ Streptococcus pyogenes ◆ Diphtheria Which one of the following antigens is involved in the entry of Plasmodium vivax into red cells? Anti-D Anti-S Duffy Kell Kidd Duffy Constipation, mucopurulent anal discharge, perianal pruritis and anal bleeding and pain while opening bowels in homosexual man ? Think of Gonorrhoea (Smear reveals intracellular diplococci). Risk of vertical transmission in Fully controlled HIV ? perinatal transmission<1% and most likelihood of transmission is during delivery (risk minimized by intrapartum zidovudine and 6 weeks after birth to baby). MRCP Infectious Disease Revision notes Dr.Sherif Elbadrawy
  • 58. Clinical picture of Tapeworm infection ? ✼ Tape worms are made up of repeated segments called proglottids (actively motile, elongating and contracting) ✼ often present in faeces and are useful diagnostically ✼ Taenia solium (pork tapeworm) and Taenia saginata (beef tapeworm) infection『Cysticercosis』 ✼ niclosamide or praziquantel. The larvae of which one of the following helminth species undertakes a symptomatic travel through the LUNG before the adult worms reside in the intestine? Enterobfus vermfcularfs Trfchfnella spfralfs Ascaris lumbricoides Trfchurfs trfchurfa Taenfa sagfnat Ascaris lumbricoides Rx of choice for cryptococcus neoformans ? Amphotrecin B Definition of Multidrug-resistant tuberculosis ? ✪ resistance to rifampicin and isoniazid, with or without resistance to other anti-TB drugs. ✪ Rx at least 3 drugs to which the organism is sensitive in-vitro for at least 9 months and perhaps up to or beyond 24 months Rx of Multidrug-resistant tuberculosis ? ✪ has to be planned on an individual basis ✪ should only be carried out by physicians with substantial experience in managing complex resistant cases ✪ at least 3 drugs to which the organism is sensitive in-vitro for at least 9 months and perhaps up to or beyond 24 months MRCP Infectious Disease Revision notes Dr.Sherif Elbadrawy
  • 59. percentages of untreated pts that will develop late stages CNS & CVS complications ? 80% 90% 10% 30% 0% 30% Difference Dengue and Malaria Fever ? ✷ Both spread by MOSQUITOES. Malaria is dt a parasite plasmodium whereas dengue is dt a flavivirus (Dengue virus) ✷ Dengue is the fever é sudden onset and remains for long duration. It is reaches to the temperature level of ≅ 40 ℃ ✷ SEVERE HEADACHE mostly in the back portion of the eyes head is CHARACTERISTIC FOR DENGUE. ✷ The dengue fever may disappear after the few days but sudden rise again with the problem of SKIN RASHES (rash of Dengue isn't dependant on thrombocytopenia)(similar to measles) ✷ in malaria, fever is for shorter period. The symptoms of malaria are anemia, pain in the joints, vomiting, sweating etc. There are 3 stages for the malaria symptoms. They are Cold stage, Sweating stage and the last is again cold stage. ✷ Incubation period 10-15 days for malaria & 4-5 days for dengue. in Dengue Lymphocytosis is seen although the total WBCs may be normal. Selective serotonin reuptake inhibitor (SSRI) withdrawal ? SSRI withdrawal appears to be a particular problem with paroxetine, which can lead to symptoms of psychomotor agitation, poor sleep and anxiety, vertigo and light headedness. ± GIT symptoms. avoided by slow tapering down the drug. Dx of Schistosoma Hematobium ? a mid-day urine sample ➜ look for the parasite Clinical picture of Acute schistosomiasis (Katayama's fever) ? may occur weeks after the initial infection, especially by S. mansoni and S. japonicum. Manifestations include: Abdominal pain Cough Diarrhea ↟ Eosinophilia Fever Fatigue Hepatosplenomegaly MRCP Infectious Disease Revision notes Dr.Sherif Elbadrawy
  • 60. Elderly + orbital cellulitis +maxillary sinusitis + C&S ➜ irregularly branching septate hyphae ,Dx ? Aspergillus Clinical picture of Mucor mycosis ? DM is the ppt factor. Rhinocerebral Mucor mycosis is the most common form. necrotic black nasal turbinate, nasal stufiness,facial pain & edema are characteristic. 35 sailor ,painless swelling in the sole of his foot , increase in size,ulceration + yellow, white grains on the surface Gm stain ➜Gm +ve branching organism Nocardia brasiliens Mucocutaneous ulceration following travel? Leishmania brasiliensis Start anti-retrovirals in HIV when CD4 < 350 MRCP Infectious Disease Revision notes Dr.Sherif Elbadrawy
  • 61. Management of contacts of meningococcal meningitis ciprofloxacin is the drug of choice as it is widely available and only requires one dose, rifampicin may be used Management of contacts of pneumococcal meninigitis no prophylaxis is generally needed Management of Meningococcal Meningitis pre-hospital (for example a GP surgery) and meningococcal disease is suspected then intramuscular benzylpenicillin , as long as this doesn't delay transit to hospital ➜ If immediate hypersensitivity reaction to penicillin or to cephalosporins ➜ Give Chloramphenicol. Rx of Cellulitis ✯ Penicillin is the antibiotic of choice for group A streptococcal infections. ✯ flucloxacillin is first-line treatment for mild/moderate cellulitis. [ stop flucloxacillin if streptococcal infection is confirmed in patients with cellulitis, dt the high sensitivity & change it with Penicillin] ✯ Clarithromycin or clindamycin is recommend in patients allergic to penicillin. Brucellosis ✯ > in Middle East and in farmers ✯ fever, malaise ✯ leukopenia ✯ hepatosplenomegaly ✯ sacroilitis: spinal tenderness ✯ osteomyelitis, infective endocarditis, orchitis, meningoencephalitis ✯ screening ➜ Rose Bengal plate test ✯ Brucella serology is the best test for diagnosis ✯ Rx ➜ doxycycline and streptomycin MRCP Infectious Disease Revision notes Dr.Sherif Elbadrawy
  • 62. Organisms which may colonise CF patients ✯ Staphylococcus aureus ✯ Pseudomonas aeruginosa ✯ Burkholderia cepacia ➜ Pseudomonas cepacia ✯ Aspergillus Bacteriostatic antibiotics ✯ chloramphenicol ✯ macrolides ✯ tetracyclines ✯ sulphonamides ✯ trimethoprim Epididymo-orchitis ✯ infection of the epididymis ± testes ➜ pain and swelling ✯ dt local spread of infections from the genital tract (Chlamydia trachomatis and Neisseria gonorrhoeae) or the bladder. ✯ U/L testicular pain and swelling ±urethral discharge ✯ DD is testicular torsion ➜ Pt < 20 years, severe pain and acute onset ✯ Rx ➜ IM ceftriaxone stat + oral doxycyline for 2 weeks HIV drugs, rule of thumb: NRTIs end in 'ine' Pis: end in 'vir' NNRTIs: nevirapine, efavirenz MRCP Infectious Disease Revision notes Dr.Sherif Elbadrawy